NBME 10 Unsolved Questions
NBME 10 Unsolved Questions
An otherwise healthy 11-year-old girl is brought to the physician by her mother because of a
3-month history of right breast tenderness. She is at the 50th percentile for height and weight.
Breast development is Tanner stage 1 on the left and Tanner stage 2 on the right. The left breast
is flat and nontender, and the right breast
is slightly raised, enlarged, and tender, there is no nipple discharge. There is no axillary or pubic
hair. Pelvic examination shows normal-appearing external genitalia. Which of the following is the
most appropriate next step in management?
OA) Reassurance
B) Measurement of serum prolactin concentration
C) Ultrasonography of the pelvis
D) CT scan of the pituitary fossa
E) Biopsy of the breast
2. A 13-year-old girl is brought to the physician by her mother because of a 10-day history of
vaginal spotting. This is her first episode of vaginal bleeding. She has never been sexually
active. She has mild asthma treated with an inhaled B-adrenergic agonist as needed. She is at
the 75th percentile for height and 80th percentile
for weight and BMI. Breast and pubic hair development are Tanner stage 4. The external
genitalia are normal. The remainder of the examination shows no abnormalities. Which of the
following is the most appropriate next step in management?
OA) Follow-up examination in 6 months
B) Measurement of serum follicle-stimulating hormone concentration
C) Vaginal ultrasonography
OD) CT scan of the head
OE) Oral contraceptive therapy
3. A7-month-old boy is brought to the physician because of a 4-day history of persistent fever.
He was born at term following an uncomplicated pregnancy and delivery; there was prolonged
bleeding following circumcision. At the age of 3 months, he developed eczema over the face
and elbows that now involves the entire
body. He has also had frequent episodes of thrush and otitis media and was recently discharged
following treatment of pneumococcal bacteremia. His current temperature is 38.5°C (101.3°F),
pulse is 120/min, and respirations are 32/min. Examination shows a diffuse eczematoid rash
with areas of purulent drainage.
Cervical lymph nodes are movable and range in size from 1.5 to 2 cm. There are white plaques
over the oral cavity. The liver edge and spleen tip are palpated 2 cm below the costal margin.
Laboratory studies show:
Hemoglobin
10 g/dL
Leukocyte count
5000/mm3
Segmented neutrophils
60%
Lymphocytes
30%
Monocytes
10%
Platelet count
35,000/mm3
Serum
IgA
100 mg/dL
IgG
400 mg/dL
IgM
15 mg/dL
IgE
60 IU/mL
Which of the following is the most likely diagnosis?
OA) Adenosine deaminase deficiency
B) Ataxia telangiectasia
C) HIV infection
D) Hyper-IgE syndrome
E) Thymic-parathyroid dysplasia (DiGeorge syndrome)
F) Wiskott-Aldrich syndrome
G) X-linked agammaglobulinemia
H) X-linked lymphoproliferative disease
1) X-linked severe combined immunodeficiency
4. A 59-year-old woman with a 5-year history of ulcerative colitis comes to the office to discuss
pharmacotherapy options. Her condition has been progressively less responsive to oral
prednisone therapy. She also has Ehlers-Danlos syndrome and asthma. Other medications are
mesalamine, albuterol, and tiotropium. The
patient is allergic to sulfonamide drugs. She appears pale and has cushingoid facies, she is not
in acute distress. Temperature is 36.9°C (98.4°F), pulse is 90/min, respirations are 16/min, and
blood pressure is 110/60 mm Hg. Lungs are clear to auscultation. Cardiac examination
discloses a grade 2/6 systolic ejection murmur
heard best at the upper left sternal border; the murmur does not radiate. Abdomen is mildly
distended and tender to palpation. There is 1+ edema of the lower extremities. Results of
laboratory studies are shown:
Blood
Hematocrit
36%
Hemoglobin
11.0 g/dL
WBC
13,000/mm3
Neutrophils, segmented
60%
Eosinophils
1%
Lymphocytes
36%
Monocytes
2%
Basophils
1%
Platelet count
197,000/mm3
The most appropriate pharmacotherapy for the patient is being considered. Which of the
following is most appropriate for this patient to undergo before beginning this medication?
A) Chest x-ray
B) Dental examination
OC) Echocardiography
D) Sulfonamide desensitization
O E) Tuberculosis screening
5. A 27-year-old man with sickle cell disease is admitted to the hospital because of a 2-day
history of increasingly severe pain in his shoulders, back, and shins. During the past 6 months,
he has been hospitalized three times for treatment of painful crises unresponsive to outpatient
therapy with hydrocodone. His temperature is
37.2°C (99°F), pulse is 90/min, respirations are 18/min, and blood pressure is 108/52 mm Hg.
The lungs are clear to auscultation. Examination shows tenderness over the shoulders, back,
and shins. His hematocrit is 30%, and reticulocyte count is 11%. In addition to pain control,
which of the following is the most appropriate
next step in management?
OA) Bone scan
B) Antibiotic therapy
OC) Intravenous administration of fluids
D) Blood transfusion
OE) Plasmapheresis
6. A 72-year-old man comes to the emergency department because he has vomited blood three
times during the past 12 hours. He has had dizziness during this period. He has a 5-year history
of arthritis treated with daily ibuprofen. Over the past 4 weeks, he has increased his daily dose
of ibuprofen because of worsening joint
pain but is not sure how many pills he has been taking. He has been unable to take his daily
walks because of the joint pain. He has not had chest pain. He had an uncomplicated
myocardial infarction 10 years ago. He appears pale. His temperature is 37.5°C (99.5°F), pulse
is 110/min, respirations are 24/min, and blood
pressure is 90/60 mm Hg. Examination shows dried blood near the mouth. The lungs are clear
to auscultation. Heart sounds are normal. The stool is black, and test for occult blood is positive.
His hematocrit is 22%, and platelet count is 215,000/mm2 Upper endoscopy shows
hemorrhagic gastritis with no active site of
bleeding. Thirty minutes after fluid resuscitation
with crystalloid solution and transfusion of 4 units of crossmatched
packed red blood cells, the patient has shortness of breath. Diffuse rhonchi and crackles are
heard bilaterally. Which of the following is the most likely cause of the dyspnea?
A) ABO incompatibility
B) Acute respiratory distress syndrome
C) Aspiration of gastric contents
D) Fluid overload
E) Pulmonary embolism
7. Six hours after admission to the hospital for management of a seizure secondary to
hyponatremia, a 53-year-old man with schizophrenia is found to have iron deficiency anemia.
He says he has been drinking 1 gallon of water daily during the past 2 months. He is receiving
haloperidol in the hospital. Physical examination
shows no abnormalities. His fluid balance has normalized. His psychosis has improved, and he
consents to colonoscopy, which shows a large tumor in the ascending colon. When the
physician mentions cancer, the patient immediately says, “I don't have cancer; I stopped
drinking a lot of water after I was admitted to the
hospital." Which of the following is the most likely explanation for this patient's response?
A) Dementia
OB) Denial
C) Rationalization
D) Reaction formation
OE) Regression
8. A 25-year-old woman comes to the physician because of a 6-month history of itching over her
arms that has become worse during the past 2 months. She attributes the itching to small
worms, which she reports having seen appear from under her skin intermittently. On several
occasions, she has "dug them out" with a clean
needle. She has brought a small box containing samples of her dry skin and debris. She reports
no history of depressed mood or auditory hallucinations. There is no personal or family history
of serious illness. The patient takes no medications and does not use illicit drugs. Vital signs are
within normal limits. Physical
examination shows mildly inflamed lesions over the upper extremities with scabbing and
excoriations. Examination of the samples she provides shows skin, scabs, and cotton fibers. On
mental status examination, she has an anxious mood and constricted affect. Her speech is
normal in rate and tone, and her thought
processes are organized and coherent. She insists that she has parasites in her skin. Which of
the following is the most likely diagnosis?
A) Bipolar disorder
OB) Delusional disorder
C) Generalized anxiety disorder
D) Major depressive disorder with psychotic features
E) Schizophrenia
9. A 15-year-old boy is brought to the physician by his parents for an examination prior to
participating on the school basketball team. Five weeks ago, he had a 1-week history of fever,
sore throat, and diffuse muscle aches. Since then, he has had decreased energy and needs at
least 10 hours of sleep every night. While the
patient's parents are out of the room, the patient states that he does not want to play basketball,
but his parents are forcing him to join the team. He is an honor student but failed an
examination last week. He does not drink alcohol, but he smokes marijuana occasionally; he
does not use any other illicit drugs. He has been
sad and angry since his girlfriend broke up with him 3 weeks ago after she told him that he was
"too moody and not fun to be around anymore." His pulse is 88/min, and blood pressure is
120/72 mm Hg. Physical examination shows no abnormalities. On mental status examination,
he has a sad mood and a full range of
affect. Which of the following is the most appropriate next step in management?
OA) Reassurance
B) Assessment for suicide risk
OC) Serologic testing for Epstein-Barr virus
D) Thyroid function tests
E) Urine toxicology screening
10. A 16-year-old boy is brought to the emergency department by his mother 30 minutes after a
4-minute episode of generalized shaking of his body. One hour ago, the patient's friend brought
him home after a party, at which time, the patient exhibited unusual behavior and said he was a
superhero with the ability to fly. The
patient subsequently became unresponsive and the episode began. The friend reports that he
saw the patient "smoke weed" at the party 2 hours before the episode. On arrival, the patient is
diaphoretic and difficult to arouse. His temperature is 37.0°C (98.6°F), pulse is 120/min,
respirations are 24/min, and blood pressure
is 135/95 mm Hg. Pulse oximetry on room air shows an oxygen saturation of 99%. The pupils
are 6 mm and sluggishly reactive to light. There is horizontal nystagmus. Use of which of the
following substances is the most likely cause of this patient's symptoms?
OA) Diazepam
B) Ethanol
OC) Marijuana
D) Methamphetamine
OE) PCP (phencyclidine)
11. A 38-year-old woman is brought to the emergency department by her husband 30 minutes
after she sustained a generalized tonic-clonic seizure. Her husband reports that she has had
worsening headache during the past 2 days and had the gradual onset of confusion 1 hour
prior to the seizure. The patient has no history of serious illness. Her only medication is an oral
contraceptive. She has smoked one pack of cigarettes daily for 20 years. On arrival, she is
sleepy but arousable. Her temperature is 37.8°C (100°F). Physical examination shows no
abnormalities. On mental status examination, she is oriented to person but not to place or time.
She is distractible and recalls zero of three objects after 5 minutes. A T2-weighted MRI of the
brain is shown. Which of the following is the most appropriate next step in diagnosis?
A) Brain biopsy
B) Cerebral angiography
UC) EEG
D) Intracranial MR venography
E) Lumbar puncture
10020
12. A 54-year-old woman is brought to the emergency department because she was unable to
move her right side or speak when she awoke this morning. Her husband reports that he found
her in bed with these symptoms 25 minutes ago, when he returned home from working an
overnight shift. He says she understood what
he was saying to her but was unable to respond. He says she felt well the night before. He adds
that she fainted three times during the past 2 years. She has no history of serious illness and
takes no medications. She appears to be in distress. She seems to understand what is said to
her but is unable to respond. She is
165 cm (5 ft 5 in) tall and weighs 82 kg (180 lb); BMI is 30 kg/m2 Her temperature is 36.4°C
(97.5°F), pulse is 80/min and regular, respirations are 12/min, and blood pressure is 142/89 mm
Hg. Examination shows weakness of the right lower aspect of the face. Muscle strength is 3/5 in
the right extremities. Babinski sign is
present on the right. The remainder of the examination shows no abnormalities. MRI of the brain
shows an infarct in the anterior branches the left middle cerebral artery. Which of the following is
the most appropriate pharmacotherapy?
A) Alteplase
B) Aspirin
C) Dabigatran
D) Heparin
OE) Warfarin
13. A previously healthy 47-year-old man comes to the emergency department because he has
been unable to close his right eye during the past 4 hours. He noticed the problem while
shaving this morning. During breakfast, he had difficulty closing his mouth to chew and could not
taste his food. His temperature is
37.2°C (996F), pulse is 84/min, respirations are 16/min, and blood pressure is 132/80 mm Hg.
Examination of the face shows a loss of the right labial fold. There is no movement of the right
side of the mouth when the patient is asked to smile, and he is unable to wrinkle the forehead
on the right. The remainder of the
examination shows no abnormalities. Which of the following is the most appropriate next step in
diagnosis?
A) CT scan of the head with contrast
B) MRI of the cervical spine
OC) Lumbar puncture
D) Electromyelography and nerve conduction studies
OE) No further testing is indicated
14. A 32-year-old woman is brought to the emergency department because of a 1-day history of
worsening confusion. Two months ago, she underwent successful gastric bypass for morbid
obesity, but she has not adhered to her physician's instructions for health maintenance since
she was discharged from the hospital. She
has been taking no regular medications or supplements and has not followed the recommended
diet. She has smoked one pack of cigarettes daily for 15 years, and she drinks two glasses of
wine daily. On arrival, she is oriented to person but not to place or time. Vital signs are within
normal limits. Examination shows
disconjugate gaze and marked nystagmus. Muscle strength is normal bilaterally. She cannot
stand with her feet together, she stands unsteadily with her feet 12 inches apart. Mental status
examination shows moderately impaired attention. She is admitted to the hospital, and medical
and physical therapy are begun. Her
condition improves, and she is reexamined 1 week later in preparation for discharge home. The
patient agrees to take all necessary medications and follow the recommended diet. Which of the
following is the most appropriate next step to prevent future similar episodes in this patient?
A) Abstinence from alcohol
B) Adherence to a low-fat diet
C) Adherence to a low-protein diet
D) Vitamin B1 (thiamine) supplementation
E) Vitamin B12 (cyanocobalamin) supplementation
Correct Answer: D
15. An 8-year-old girl is brought to the emergency department by her father because of swelling
of the right side of her neck. The father first noticed a small lump on her neck last week and
says it seems larger today; it also has become discolored and now resembles a bruise. The
patient has not had fever or changes in energy
or appetite. She has had no recent infections or contact with anyone known to be ill. The family
has not traveled outside of the United States, but they spend weekends on the patient's
grandparents' ranch. The patient is at the 50th percentile for height, weight, and BMI. Vital signs
are within normal limits. She appears well.
Physical examination shows a 3 * 5-cm firm, mobile mass in the right anterior cervical chain
region, there is no erythema and the mass is minimally tender. A purplish hue is noted over the
mass. There is no other lymphadenopathy, and no hepatomegaly or splenomegaly is noted.
Excisional biopsy specimen of the mass
shows the presence of multiple granulomas. Which of the following infectious agents is the most
likely cause of this patient's condition?
A) Cytomegalovirus
B) Epstein-Barr virus
OC) Haemophilus influenzae, type b
D) Mycobacterium avium-intracellulare
E) Staphylococcus aureus
16. A 55-year-old woman with a 10-year history of type 2 diabetes mellitus returns to the clinic
for a follow-up examination of a right foot ulcer diagnosed 8 weeks ago. At that time, she was
treated with hydrocolloid dressings. Today, the patient reports a 1-week history of chills and
moderate mid back pain. She has not had
cough, pain with urination, or diarrhea. She also has hypertension. Medications are metformin
and lisinopril. The patient is 160 cm (5 ft 2 in) tall and weighs 88 kg (195 lb); BMI is 35 kg/m2
Her temperature is 38.2°C (100.8°F), pulse is 90/min, respirations are 18/min, and blood
pressure is 132/80 mm Hg. Examination
shows a stage 2 foot ulcer that measures 2 cm in diameter and point tenderness over the T12
spinous process. The remainder of the examination shows no abnormalities. Which of the
following studies is most likely to confirm the cause of this patient's fever and back pain?
OA) Blood cultures
B) Determination of erythrocyte sedimentation rate
C) Echocardiography
D) MRI of the thoracic spine
E) Ultrasonography of the abdomen
17. A 67-year-old woman comes to the physician because of a 1-week history of moderate pain
and swelling, redness, and warmth of her right knee. She has a 10-year history of moderate
pain and progressive stiffness of her hands and a 1-year history of swelling and warmth of her
digits and wrists. She says she has
generalized stiffness for 1 hour after awakening in the morning. She has no history of trauma to
the hands or knees. She has no history of serious illness and takes ibuprofen for the pain as
needed. Vital signs are within normal limits. On examination, the metacarpophalangeal joints
are boggy bilaterally and tender on
squeezing. There is erythema and swelling of the right knee; the patella is ballotable. Which of
the following is the most appropriate next step in management?
OA) X-ray of the right knee
B) Oral naproxen therapy
C) Oral oxycodone therapy as needed for pain
D) Oral prednisone therapy
O E) Arthrocentesis of the right knee
18. A 22-year-old woman comes to the office because of a 6-week history of mild pain and
swelling in the front of her right ankle. She says her symptoms began after she awkwardly
stepped off a curb. There is no personal or family history of serious illness. Her only medication
is ibuprofen as needed for pain. She does not
smoke cigarettes or drink alcohol. Vital signs are within normal limits. Examination shows a
1-cm abrasion and mild ecchymosis over the anterolateral aspect of the right ankle. There is
warmth and swelling at the right anterior ankle joint, passive dorsiflexion produces pain. Anterior
drawer test is negative bilaterally. Muscle
strength is 5/5 on inversion, eversion, dorsiflexion, and plantar flexion of the right ankle. Dorsalis
pedis pulses are 2+ bilaterally. Sensation to light touch is intact over the lower extremities.
X-rays of the right ankle are shown. Which of the following is the most appropriate next step in
diagnosis?
A) Bone biopsy
OB) Complete blood count
C) Joint aspiration of the right ankle
D) MRI of the right tibia
E) Ultrasonography of the right lower extremity
Correct Answer: D.
19. A 16-year-old girl is brought to the physician because of generalized weakness for the past
3 days. She states that during the past month, she has been following a strict diet and taking
laxatives and diuretics to lose weight. She has no history of serious illness. Vital signs are within
normal limits. Examination shows diffuse,
generalized weakness; there is no pain on muscle palpation. Deep tendon reflexes are 1+ in the
upper and lower extremities. A decreased concentration of which of the following serum
electrolytes is the most likely cause of this patient's muscle weakness?
A) Calcium
B) Chloride
OC) Magnesium
OD) Potassium
E) Sodium
20. A61-year-old man with ischemic cardiomyopathy is admitted to the hospital for management
of community-acquired pneumonia. His medications are furosemide, spironolactone, and
lisinopril. He has a dietary sodium restriction of less than 1500 mg daily. His temperature is
38.8°C (101.8°F), pulse is 100/min, respirations
are 22/min, and blood pressure is 128/74 mm Hg. Pulse oximetry on room air shows an oxygen
saturation of 88%. Crackles are heard in the middle lobe of the right lung.There is no edema of
the lower extremities. A chest x-ray shows a right middle lobe infiltrate. His ejection fraction is
30% Which of the following is the
most appropriate intravenous fluid maintenance therapy?
A) 5% Dextrose in 0.45% saline
B) 5% Dextrose in 0.9% saline
C) 5% Dextrose in water
D) 0.9% Saline
OE) No intravenous fluids are indicated
21. A 64-year-old woman is brought to the emergency department because of a 3-hour history
of shortness of breath, nausea and vomiting. She has not had fever or chest or abdominal pain.
She has not had any sick contacts. She has hypertension and type 2 diabetes mellitus. Her
medications are lisinopril, insulin, and
aspirin. On arrival, she is in acute distress. Her pulse is 64/min with occasional premature beats,
respirations are 22/min, and blood pressure is 90/50 mm Hg. The lungs are clear to
auscultation. On cardiac examination, S, and Szare normal and an Seis heard; there are no
murmurs. The abdomen is soft and nontender.
There is no peripheral edema. An ECG is shown. Chest x-ray shows no pulmonary edema.
Administration of which of the following is contraindicated in this patient?
OA) Heparin
B) Nitroglycerin
C) 0.9% Saline
D) Simvastatin
E) Streptokinase
22. A 57-year-old woman comes to the physician as a new patient. She has type 2 diabetes
mellitus that was poorly controlled with glyburide for 10 years; it has been better controlled with
the addition of metformin for the past 2 years. She does not smoke cigarettes or drink alcohol.
She is 152 cm (5 ft) tall and weighs 63 kg
(140 íb); BMI is 27 kg/m2 Her blood pressure is 165/102 mm Hg on three measurements. One
year ago, her blood pressure was 144/90 mm Hg. Examination today shows no abnormalities.
Serum studies show electrolyte concentrations within the reference ranges and a creatinine
concentration of 0.9 mg/dL. Urinalysis
shows 4+ protein. Which of the following is the most likely mechanism of this patient's increased
blood pressure?
A) Aldosterone-secreting adrenal tumor
OB) Atherosclerosis of the renal arteries
C) Epinephrine-secreting adrenal tumor
D) Renal parenchymal disease
E) Renin-secreting adrenal tumor
23. A 25-year-old woman comes to the office for a health maintenance examination. She is on
active duty in the US Air Force, and her primary duty is flying drones. She reports no symptoms.
Medical history is unremarkable, and she takes no medications. She does not smoke cigarettes
or drink alcoholic beverages. Her
paternal grandmother had a myocardial infarction at the age of 62 years. Vital signs are within
normal limits. Physical examination discloses no abnormalities. Results of fasting serum lipid
studies obtained 1 year ago are shown:
Cholesterol
Total
205 mg/dL
HDL-cholesterol
45 mg/dL
LDL-cholesterol
150 mg/dL
Triglycerides
50 mg/dL
Which of the following is the most appropriate screening laboratory study at this time?
A) LDL-receptor activity assay
B) Measurement of serum apolipoprotein(a) concentration
C) Measurement of serum C-reactive protein concentration
D) Repeat measurement of fasting serum lipid studies
OE) No further testing is indicated
24. A 57-year-old man comes to the physician because of a 3-month history of an open wound
on the left leg; the wound has been increasing in size during this time. He also has had mild
pain and the sensation of heaviness in the entire left leg during this time, the pain is worst at the
end of the
day and improves with leg elevation. His medications are inhaled albuterol as needed for
asthma and acetaminophen as needed for the leg pain. He has smoked one pack of cigarettes
daily for 40 years. He lives in the southwestern USA and works as a long-distance truck driver.
Vital signs
are within normal limits. A photograph of the left leg is shown. The left ankle brachial index is
0.75 (N>1). The remainder of the examination shows no abnormalities. Which of the following is
the most likely cause of these findings?
OA) Brown recluse spider bite
B) Malignant melanoma
C) Peripheral artery disease
D) Squamous cell carcinoma
E) Venous insufficiency
25. A previously healthy 4-year-old boy is brought to the physician because of a 3-day history of
fever, cough, and runny nose. He has not had wheezing, vomiting, or diarrhea. He is at the 75th
percentile for height and 10th percentile for weight. His temperature is 37.5°C (99.5°F). The skin
is warm and pink. Capillary refill time
is 2 seconds. Examination shows clear rhinorrhea. Breath sounds are normal. Cardiac
examination is shown. The remainder of the examination shows no abnormalities. Which of the
following is the most likely diagnosis?
A) Congestive heart failure
B) Idiopathic pulmonary hypertension
C) Pericarditis
D) Upper respiratory tract infection
E) Ventricular septal defect
26. An asymptomatic 37-year-old man who has been HIV positive for 1 year comes to the
physician for a follow-up examination. His CD4+ T-lymphocyte count is 650/mm3 (Normal2500).
A PPD skin test produces 6 mm of induration at 48 hours. An x-ray of the chest shows no
abnormalities. Which of the following is the most
appropriate next step in management?
OA) Repeat PPD skin test in 10 weeks
B) Repeat PPD skin test in 1 year
C) Second strength PPD skin test (250 tuberculin units) now
D) Treatment of active tuberculosis now
OE) Treatment of latent tuberculosis now
F) No further testing or therapy
27. An 18-year-old woman is admitted to the hospital because of a 3-day history of fever,
shortness of breath, and cough productive of sputum. She received the diagnosis of asthma at
the age of 12 years. She is currently in acute respiratory distress. Her temperature is 37.3°C
(99.1°F), pulse is 120/min, respirations are
30/min, and blood pressure is 120/70 mm Hg. Pulse oximetry on 4 Umin of oxygen by nasal
cannula shows a decrease in oxygen saturation from 98% on admission to 90%. Examination
shows use of accessory muscles of respiration. Breath sounds are decreased bilaterally. Her
leukocyte count is 18,000/mm3 A chest x-
ray shows hyperinflation, which is consistent with X-ray findings on admission. Which of the
following is the most likely cause of this patient's current condition?
OA) Aspiration
OB) Bronchospasm
C) Pneumonia
D) Pneumothorax
E) Pulmonary edema
OF) Pulmonary embolus
28. A 51-year-old man comes to the physician because of a 1-year history of restless sleep and
fatigue in the morning. He says that his wife has asked him to sleep in a separate bedroom
because of his excessive snoring during this time. The patient has no history of serious illness
and takes no medications. He is 191 cm (6 ft
3 in) tall and weighs 124 kg (273 lb); BMI is 34 kg/m2. His pulse is 82/min, respirations are
16/min, and blood pressure is 150/90 mm Hg. Pulmonary examination shows no abnormalities.
On cardiac examination, a loud Szis heard at the second left intercostal space. Which of the
following is most likely to be seen on
echocardiography?
A) Decreased left ventricular ejection fraction
B) Decreased right ventricular ejection fraction
OC) Increased pulmonary artery pressure
D) Increased pulmonary venous pressure
E) Left atrial hypertrophy
OF) Right atrial hypertrophy
29. A 77-year-old man comes to the office because of a 1-day history of bloody diarrhea and
moderate abdominal pain. He has had a decreased appetite and four to five bloody stools daily
during this period. He was discharged from the hospital 1 week ago following management of
an anterior wall myocardial infarction. He has
not had fever, chills, vomiting, or recent exposure to sick contacts since he left the hospital. He
has not traveled outside of the USA during the past year. His medications are enalapril,
atorvastatin, metoprolol, and daily aspirin. He appears moderately ill. His pulse is 95/min, and
blood pressure is 105/72 mm Hg. Cardiac
examination shows a grade 2/6 systolic ejection murmur. Abdominal examination shows diffuse
tenderness that is most prominent in the left lower quadrant, there is voluntary guarding but no
rebound tenderness, and no masses are palpated. Results of laboratory studies are shown:
Hematocrit
39%
Leukocyte count 12,000/mm3
Platelet count
175,000/mm3
Serum
Na+
139 mEq/L
K+
3.9 mEq/L
CI-
106 mEq/L
HCO,
22 mEq/L
Urea nitrogen 29 mg/dL
Creatinine 1.2 mg/dL
Fecal leukocytes 0/mm3
Which of the following is the most likely diagnosis?
A) Angiodysplasia
B) Colon cancer
C) Cytomegalovirus proctitis
OD) Diverticulitis
OE) Escherichia coli O157:H7 infection
F) Ischemic colitis
30. A 46-year-old woman, gravida 3, para 3, comes to the emergency department because of a
4-day history of "crampy" lower abdominal pain. She voids six times daily and once at night. Her
last bowel movement was 3 days ago. She has not had fever, nausea, vomiting, or changes in
appetite. She has hypertension,
hypothyroidism, chronic back pain, and overactive bladder. Medications are enalapril,
levothyroxine, tramadol, mirabegron, and acetaminophen. She is sexually active with her
husband. She appears mildly uncomfortable. Temperature is 37.5°C (99.5°F), pulse is 64/min,
respirations are 12/min, and blood pressure is 132/86
mm Hg. Pulse oximetry on room air shows an oxygen saturation of 98%. No abdominal
tenderness in noted. The remainder of the examination shows no abnormalities. Adverse effect
of which of the following is the most likely cause of this patient's presenting symptom?
OA) Acetaminophen
B) Enalapril
C) Levothyroxine
D) Mirabegron
O E) Tramadol
31. A 69-year-old woman comes to the physician because of a 6-day history of blood in her
urine. She has not had pain. She has no history of serious illness and takes no medications.
Vital signs are within normal limits. Examination shows no abnormalities. Urinalysis shows many
RBCs. Cystoscopy shows a 2-cm mass in the
bladder. Which of the following is the most likely diagnosis?
A) Bladder polyp
B) Leiomyosarcoma
C) Papilloma
OD) Transitional cell carcinoma
E) Vesicle calculus
32. An 18-year-old man is brought to the emergency department because of severe pelvic and
lower abdominal pain 30 minutes after he was involved in a motorcycle collision in which he
struck a parked car. He was wearing a helmet and did not have loss of consciousness. He is
alert. His pulse is 120/min, respirations are
18/min, and blood pressure is 130/50 mm Hg. The airway is clear and open, and breath sounds
are normal. The abdomen is nontender. There are ecchymoses over the pelvis and scrotum.
The pelvis is unstable. There is blood at the urethral meatus. Pulses are 2+ in all extremities. An
X-ray of the pelvis shows an
anteroposterior compression fracture. A pelvic binder is placed. Which of the following is the
most appropriate next step in management of this patient's urinary tract injury?
A) CT scan of the abdomen and pelvis
B) Cystography
C) Placement of a urinary catheter
D) Pyelography
O E) Retrograde urethrography
33. A 27-year-old woman, gravida 2, para 1, at 14 weeks' gestation comes to the physician for
her first prenatal visit. She is concerned because 2 years ago, during her first pregnancy, she
delivered a stillborn at 38 weeks' gestation 24 hours after she had noticed decreased fetal
movement. She has no history of serious illness.
Examination shows no abnormalities. Which of the following is the most appropriate next step in
management?
A) Monthly 1-hour glucose tolerance tests
OB) Nonstress testing in the third trimester
C) Fetal Doppler ultrasonography at 36 weeks' gestation
D) Amniocentesis for karyotype analysis
E) Cesarean delivery at 37 weeks' gestation
34. A 25-year-old woman comes to the physician for a follow-up examination because she
continues to have urinary frequency and pain with urination despite a 1-week course of
trimethoprim-sulfamethoxazole followed by a 1-week course of nitrofurantoin. A urine culture at
the onset of her symptoms 2 weeks ago showed
vaginal flora but no specific organism. She is sexually active and uses an oral contraceptive.
Examination shows a mildly erythematous, tender urethra. Purulent material can be expressed
from the periurethral glands. Vaginal examination shows no abnormalities, and there is no
vaginal discharge. Bimanual examination
shows a nontender cervix and uterus. The cervix is friable and bleeds easily after a Pap smear.
Which of the following is the most likely diagnosis?
A) Bartholin gland abscess
OB) Chlamydia trachomatis infection
C) Interstitial cystitis
D) Pseudomonal urinary tract infection
E) Pyelonephritis
OF) Urethral diverticulum
35. A 37-year-old woman comes to the physician for a routine follow-up examination. She has a
12-year history of type 2 diabetes mellitus treated with glipizide and diet. She has urinary
frequency when she does not follow her diet. She is otherwise healthy. There is no family history
of diabetes mellitus. Her pulse is 72/min, and
blood pressure is 130/80 mm Hg. Funduscopic examination shows scattered microaneurysms.
The remainder of the examination shows no abnormalities. Which of the following is most likely
to determine this patient's risk for diabetic nephropathy?
A) Measurement of hemoglobin A10
B) Measurement of serum albumin concentration
C) Measurement of serum glucose concentration
OD) Measurement of urine microalbumin concentration
E) Measurement of urine protein by dipstick
38. A 54-year-old woman with hypertension is admitted to the hospital because of a 1-week
history of nausea, vomiting, and diarrhea. She says she has been vomiting four to five times
daily and has not been able to keep any food or liquids down. She has had up to 10 nonbloody
loose stools daily. She reports no fever or
chills. She says she previously had been well, but she noticed that her blood pressure had been
higher than usual during the past 2 weeks, and she was planning to see her primary care
physician. She is a pediatric nurse and had been caring for several patients with unspecified
viral gastroenteritis 1 week before she
became ill. She has hyperlipidemia. Medications are hydrochlorothiazide and atorvastatin. The
patient is 168 cm (5 ft 6 in) tall and weighs 73 kg (162 lb); BMI is 26 kg/m2. Temperature is
37.3°C (99.2°F), pulse is 110/min, and blood pressure is 126/68 mm Hg. Abdominal
examination discloses mild, diffuse tenderness to
palpation without rebound or guarding. Results of complete blood count and urinalysis are within
the reference ranges. Results of serum chemistry profile and urine electrolyte concentrations
are shown:
Urine
Urea nitrogen 22 mg/dL Na
<10 mmol/L
Creatinine 1.0 mg/dL
K 30 mmol/L
Na+
136 mEq/L CI- 12 mmol/L
K:
3.1 mEq/L
CI-
95 mEq/L
34 mEq/L
HCO3
Arterial blood gas analysis on room air:
PO 98 mm Hg
Pcoz 49 mm Hg
pH 7.51
Which of the following is the most likely cause of this patient's laboratory results?
OA) Addison disease
B) Adrenal adenoma
C) Diarrhea
D) Diuretic use
OE) Vomiting
39. A case-control study is conducted to assess the relationship between childhood infection
with varicella zoster virus and lymphoma. It is most appropriate to use which of the following
methods to minimize recall bias in this study?
A) Choose cases and controls from the same population
B) Ensure that the investigators gathering exposure information are unaware of participant
outcome status
C) Form a panel of blinded investigators to assign case and control definitions
OD) Randomly select the controls from the population
E) Validate the exposure information with blood tests
40. A cohort study is conducted to evaluate the impact of fish consumption on the risk for
developing cardiovascular disease. Participants complete a questionnaire about dietary habits,
including fish consumption; responses are obtained from 2000 participants (80% response rate).
The study participants are observed for
20 years, and cases of cardiovascular disease are identified through a data registry. Study
results show:
Cases with
Cases without
Eating Fish (Frequency) Cardiovascular Disease Cardiovascular Disease Total
Never
160
1440
1600
Almost Daily
20
380
400
Total
180
1820
2000
Based on these data, which of the following best represents the attributable risk for
development of cardiovascular disease caused by lack of fish consumption in this population?
A) 0.01
B) 0.04
C) 0.05
D) 0.07
E) 0.09
41. A study is conducted to determine if there is an association between tampon use and toxic
shock syndrome (TSS). A total of 50 patients with TSS and 450 patients without TSS are
enrolled in the study. The patients are matched for age and race. All patients are interviewed to
determine if they use tampons. When analyzing
the data, which of the following is the most appropriate statistical method to use for this study?
A) Analysis of variance
OB) Chi-square test
C) Correlation coefficient
D) Mann-Whitney test
E) Regression analysis
OF) Unpaired t-test
42. A 22-year-old man is admitted to the intensive care unit after he sustained a severe head
injury in a motorcycle collision. Three days later, he remains intubated and has had no
spontaneous respirations. His temperature is 37°C (98.6°F). His pupils are fixed and dilated,
and he is unresponsive to all physical and verbal
stimuli. He has no cranial nerve reflexes, and repeated EEG shows no electrical activity. He is
receiving no medications. Toxicology screening is negative. Which of the following is the most
accurate statement regarding continuation of mechanical ventilation for this patient?
A) Mechanical ventilation may be withdrawn only if a do-not-resuscitate order has been signed
by the next of kin
B) Mechanical ventilation may be withdrawn only if the patient has a living will
OC) Mechanical ventilation may be withdrawn only with court approval
OD) Mechanical ventilation may be withdrawn without the family's permission
43. A 77-year-old woman is brought to the physician by her son because of palpitations for the
past month. She had a cerebral infarction 5 years ago and has residual partial paralysis of her
right leg. She uses an orthotic brace on her right ankle and a walker. She has fallen three times
during the past year but has not sustained
any injuries. She has a 5-year history of type 2 diabetes mellitus treated with metformin. Her
vision is 20/60 in her left eye, but she is able to read large print. Her pulse is 120/min and
irregularly irregular, and blood pressure is 135/78 mm Hg. Crackles are heard at the lung bases.
A grade 2/6 systolic murmur is heard best at
the lower left sternal border. Abdominal examination shows no abnormalities. There is 2+
edema below the knees. Neurologic examination shows unchanged hemiparesis. Laboratory
studies show:
Hematocrit
37%
Leukocyte count 4700/mm3 with a normal differential
Serum
Urea nitrogen 27 mg/dL
Creatinine 1.4 mg/dL
An ECG shows atrial fibrillation with a ventricular rate between 90/min and 140/min.
Echocardiography shows an ejection fraction of 40%. Which of the following is the most
appropriate next step in management?
A) Measurement of serum thyroid-stimulating hormone concentration
B) Tilt test
24-Hour ambulatory ECG monitoring
D) Electrophysiologic studies
O E) Placement of a cardiac pacemaker
44. A 37-year-old man comes to the physician 12 hours after the onset of vomiting and
abdominal cramps and swelling. He has had constipation for the past 4 days. He was diagnosed
with Crohn disease 7 years ago. His symptoms of diarrhea and right lower quadrant abdominal
pain have been well controlled with mesalamine
for the past 3 years. His temperature is 36°C (96.8°F), pulse is 98/min and regular, and blood
pressure is 110/70 mm Hg. Examination shows a diffusely distended, tympanitic abdomen and
visible peristalsis; high-pitched bowel sounds are heard. Řectal examination shows no stool in
the rectal vault. An x-ray of the
abdomen shows a small-bowel obstruction. Which of the following is the most likely cause of
these findings?
OA) Colon cancer
B) lleocecal fistula
C) Small-bowel adhesions
D) Small-bowel fibrotic stricture
E) Small-bowel intussusception
45. A 62-year-old man comes to the physician to discuss the results of an abdominal CT scan
that was performed 4 days ago. He is undergoing evaluation for a 2-month history of increasing
back pain and a 6.8-kg (15-lb) weight loss. An initial physical examination showed no
abnormalities. The CT scan showed a 4-cm solid
mass in the body of the pancreas, multiple defects in the liver consistent with metastases, and a
7-cm abdominal aortic aneurysm with involvement of the renal arteries and no evidence of
leakage. A CT scan-guided biopsy of the mass is performed. Examination of the specimen
shows a pancreatic adenocarcinoma. After
listening to the physician's explanation of his prognosis, the patient says, "So you think I have a
pancreatic cancer that is too advanced to remove. But what about the aneurysm? Should I have
something done about that?" Which of the following is the most appropriate recommendation
with respect to repair of this patient's
aneurysm?
A) The aneurysm should be repaired because of its large size
B) The aneurysm should be repaired only if the procedure can be performed with minimally
invasive endovascular techniques
C) The aneurysm should not be repaired because of the location of the pancreatic cancer
D) The aneurysm should not be repaired because of the patient's poor prognosis
E) The aneurysm should not be repaired because the operation would be likely to stimulate the
growth of the pancreatic cancer
46. A 52-year-old man comes to the emergency department 3 days after he sustained an injury
to his left foot while walking barefoot on the beach. He says he initially thought he only had a
minor cut, but he now has severe pain that radiates from his left foot to his groin. Ten years ago,
he underwent cadaveric liver
transplantation for hepatitis C. Current medications include mycophenolate and prednisone. On
arrival, he appears diaphoretic and ill. His temperature is 39.2°C (102.6°F), pulse is 120/min,
respirations are 24/min, and blood pressure is 110/60 mm Hg. Examination shows erythema
and hemorrhagic bullae over the left calf
and foot. The skin is warm and tender to touch from over the left mid calf to the foot. Pulses in
the distal lower extremities are equal bilaterally. His leukocyte count is 16,000/mm3 with 82%
segmented neutrophils, 15% bands, and 3% eosinophils. Results of blood cultures are pending.
In addition to intravenous administration
of antibiotics, which of the following is the most appropriate next step in management?
OA) Discontinue prednisone
B) Hyperbaric oxygen therapy
C) Intravenous immune globulin therapy
D) Surgical debridement
OE) Amputation
OF) No additional management is indicated
47. A 72-year-old man has decreased urine output 2 days after admission to the hospital for
treatment of cholecystitis. His urine output has been 15 ml/h over the past 3 hours. On
admission, results of laboratory studies were consistent with gram-negative bacteremia and
disseminated intravascular coagulation. He is currently
receiving intravenous fluids, cefoxitin, and gentamicin. His temperature is 38.5°C (101.3°F),
pulse is 110/min, respirations are 24/min, and blood pressure is 90/64 mm Hg. Abdominal
examination shows mild right upper quadrant tenderness. His serum creatinine concentration
has increased from 1.5 mg/dL 2 days ago to
3 mg/dL. This patient is most likely to have which of the following sets of urinalysis findings?
RBC
WBC
Blood
Protein
(/hpf)
(/hpf)
Casts Other Microscopic Findings
None none
none
none
hyaline
none
B)
None
none
none
none
oval fat bodies
Trace
0-5
>50
none
WBC clumps
1+
1+
0-5
0-5 pigmented granular renal tubular epithelial cells
5-10
10-20
none
eosinophils
F)
3+
>50
none
RBC
G)
none
none pigmented granular squamous epithelial cells
H)
4+
1+ 10-20 dysmorphic
none
none
none
48. A 17-year-old patient, gravida 1, para 0, aborta 1, comes to the physician for a follow-up
examination. Four weeks ago, she underwent dilatation and curettage (D&C) following a
spontaneous abortion at 13 weeks' gestation. She has had no vaginal bleeding or pelvic pain.
The patient has no history of serious illness. Her
only current medication is an oral contraceptive; she adheres to her medication regimen. Today,
pelvic examination shows no abnormalities. Prior to the D&C, her serum B-hCG concentration
was 50,326 mlU/mL. Since the D&C, serial measurements of serum B-hCG concentrations have
shown:
Week B-hCG
(mlU/mL)
18,275
8157
8356
14.589
WN -
Which of the following is the most likely diagnosis?
A) Endometritis
OB) Gestational trophoblastic neoplasia
C) Hydatidiform mole
D) Retained products of conception
OE) Septic abortion
49. A previously healthy 23-year-old man comes to the physician because of a 5-day history of
fever, malaise, and headache. He has not had nausea, vomiting, cough, sore throat, diarrhea,
pain with urination, or penile discharge. Medications include pseudoephedrine and ibuprofen for
sinus headaches. He has no known
recent exposure to illness and has not traveled recently outside the USA. He has been sexually
active with four female partners over the past year and uses condoms inconsistently. He drinks
two six-packs of beer weekly. He lives near tick-infested woods but has not noted a tick bite. He
is 183 cm (6 ft) tall and weighs 95
kg (210 lb); BMI is 29 kg/m2. His temperature is 38.3°C (101°F), pulse is 72/min, respirations
are 12/min, and blood pressure is 126/68 mm Hg. Examination shows a nonpruritic, painless,
pigmented, maculopapular rash over the palms and soles. There is diffuse lymphadenopathy.
Which of the following is most likely to
have prevented this patient's symptoms?
A) Abstinence from alcohol
B) Avoidance of ibuprofen use
C) Avoidance of soaps containing perfume
D) Consistent use of condoms
E) Immediate removal of ticks
50. A 30-year-old primigravid woman with systemic lupus erythematosus comes to the physician
for a prenatal visit at 34 weeks' gestation. Examination shows that her disease is in remission.
She is concerned about the possible effects of her disease on her fetus. The fetus is at greatest
risk for which of the following?
A) Atrial septal defect
OB) Complete atrioventricular block
C) Endocarditis
D) Supraventricular tachycardia
E) Ventricular septal defect
4. A 9-year-old girl is brought to the physician by her parents because of increasingly strange
behavior during the past 6 months. They report that she touches and smells household objects
and repeatedly turns the lights on and off in her room. She has been worried because of
frequent thoughts of her dog being hit by a car
and her parents being involved in a motor vehicle collision even though these things have never
happened. She has begun to tap her foot three times when she experiences these thoughts to
prevent them from happening. Physical examination shows no abnormalities. Mental status
examination shows an anxious mood and
a constricted affect. Which of the following is the most appropriate pharmacotherapy?
OA) Clonidine
B) Clozapine
C) Dextroamphetamine
D) Haloperidol
E) Naloxone
F) Nortriptyline
G) Sertraline
5. A 44-year-old man comes to the office with his girlfriend because she is concerned about his
mood. He recently told her it would be a relief if he were hit by a truck and died. He reports
frequent fatigue and says he has been persistently unhappy for 9 years. He has not had
problems with sleep or appetite. He has type 1
diabetes mellitus, chronic kidney disease, hypertension, and peripheral neuropathy. His
medications are furosemide, gabapentin, insulin, metoprolol, and simvastatin. He lost joint
custody of his two children 1 month ago, which he says is "a real downer." He finds his work as
a convenience store clerk tedious and
uninteresting, but he enjoys playing poker and watching movies. He has used cocaine once
monthly during the past 5 years. His pulse is 84/min, and blood pressure is 146/88 mm Hg.
Physical examination shows retinal hemorrhages. On mental status examination, he has a
dysphoric mood and sarcastic affect. There is
evidence of interpersonal friction with his girlfriend. He reports no current suicidal ideation or
intent. Which of the following is the most likely diagnosis?
A) Adjustment disorder with depressed mood
B) Borderline personality disorder
C) Major depressive disorder
D) Persistent depressive disorder (dysthymia)
E) Substance-induced mood disorder
6. A 14-year-old boy is brought to the physician by his parents because he has been
increasingly irritable, disrespectful, and uncooperative during the past 4 years. He refuses to do
any household chores and does not obey curfew. He may be expelled from school because he
is rude and threatening to teachers and has been
cutting classes. He is tutored for reading but earns C's and D's in his classes. He has been
suspended three times during the past year for talking back to his teachers. He repeated first
grade due to poor academic readiness. He used to have trouble with homework but now he
does not complete any homework. He is at the
65th percentile for height and 50th percentile for weight. Physical examination shows no
abnormalities. During the examination, the patient is minimally cooperative and frequently rolls
his eyes at questions. Urine toxicology screening is positive for nicotine. Which of the following
is the most likely explanation for this patient's
behavior?
A) Attention-deficit/hyperactivity disorder
B) Borderline cognitive functioning
C) Conduct disorder
D) Oppositional defiant disorder
E) Reading disorder
OF) Normal adolescent behavior
7. A 72-year-old man comes to the physician because of a 2-year history of increasingly severe
episodic pain in both legs. The pain is triggered by walking and increases as he walks longer
distances. He also has weakness and numbness in both legs when he walks farther than 4
blocks. He says that he now has difficulty
walking through the grocery store, but the pain decreases when he leans forward on the cart
while walking. He has no history of serious illness and takes no medications. Vital signs are
within normal limits. Muscle strength and deep tendon reflexes are normal, and sensation is
intact. Femoral, posterior tibial, and pedal
pulses are equal bilaterally. The remainder of the examination shows no abnormalities. Which of
the following is the most likely cause of this patient's symptoms?
A) Lower extremity arterial insufficiency
B) Lower extremity venous insufficiency
C) Lumbar degenerative joint disease
D) Lumbar disc herniation
OE) Lumbar spinal stenosis
8. An 11-year-old boy is brought to the office by his mother as a new patient because of a 3-year
history of increasing weakness of his arms and legs. His mother says the weakness is now
interfering with her son's ability to participate in physical education classes at school. He has no
history of serious illness and receives no
medications. His older sister is healthy. He has several male and female cousins who are
uncoordinated and fall frequently. Examination of the patient shows scoliosis of the thoracic and
lumbosacral spine. There is atrophy and muscle weakness of both shoulders and pelvic girdles.
Deep tendon reflexes are trace throughout.
Sensation is intact throughout. His gait is waddling, and he must extend his back to walk
smoothly. Results of a complete blood count and serum electrolyte concentrations are within the
reference ranges. His serum creatine kinase activity is 387 U/L. Which of the following is the
most likely diagnosis?
A) Duchenne muscular dystrophy
B) Juvenile spinal muscular atrophy
C) Myasthenia gravis
OD) Myotonic dystrophy type 2
E) Polymyositis
9. A 32-year-old woman comes to the office for a health maintenance examination. She feels
well. She has mild myopia and wears contact lenses. She has no history of serious illness and
takes no medications. She appears well. Vital signs are within normal limits. A
photograph of the right eye is shown. The remainder of the examination shows no
abnormalities. Which of the following is the most appropriate next step in management?
OA) Incision and drainage
OB) Recommendation that she wear sunglasses
C) Serum antinuclear antibody assay
D) Surgical excision
OE) Use of antibiotic eye drops
10. A 4-year-old boy is brought to the physician because of a lesion on his scalp for the past 6
weeks. It has continued to increase in size despite use of over-the-counter antibacterial creams
and 7 days of oral cephalexin that his mother had leftover from a previous illness. The lesion
began as a round area approximately the
size of a quarter. He has no history of serious illness. A photograph of the lesion is shown.
Infection with which of the following is the most likely cause of this patient's lesion?
OA) Microsporum gypseum
B) Mycobacterium ulcerans
C) Pseudomonas aeruginosa
D) Staphylococcus aureus
E) Streptococcus pyogenes (group A)
10. A 4-year-old boy is brought to the physician because of a lesion on his scalp for the past 6
weeks. It has continued to increase in size despite use of over-the-counter antibacterial creams
and 7 days of oral cephalexin that his mother had leftover from a previous illness. The lesion
began as a round area approximately the
size of a quarter. He has no history of serious illness. A photograph of the lesion is shown.
Infection with which of the following is the most likely cause of this patient's lesion?
OA) Microsporum gypseum
B) Mycobacterium ulcerans
C) Pseudomonas aeruginosa
D) Staphylococcus aureus
E) Streptococcus pyogenes (group A)
11. A 67-year-old man comes to the emergency department because of a 2-day history of fever
and progressive pain in his right knee. He says that he had a cold 5 days ago. He has
end-stage renal failure secondary to type 2 diabetes mellitus and has required hemodialysis for
the past 2 years. Current medications include
insulin, erythropoietin, and a daily multivitamin. His temperature is 37.9°C (100.2°F). The right
knee is swollen and warm. Range of motion is severely limited due to pain. The remainder of
the examination shows no abnormalities. Which of the following is the most likely cause of this
patient's current symptoms?
OA) Hematogenous infection
B) Hemochromatosis
C) Human leukocyte antigen-B27 haplotype
D) Sequelae of a viral upper respiratory tract infection
OE) Uremia
12. A 62-year-old man comes to the physician because he has noticed bloody urine
intermittently during the past 2 months. He occasionally passes large clots through his urethra.
During the past 2 weeks, he has had pain and swelling over his distal radial forearm.
Examination shows tenderness, erythema, and edema over
the radius 10 cm proximal to the wrist joint. An X-ray of the left upper extremity shows a lytic
lesion of the radius with a pathologic fracture. Which of the following is the most likely
diagnosis?
A) Giant cell tumor
B) Leukemia of bone
C) Malignant fibrous histiocytoma
OD) Metastatic disease
E) Multiple myeloma
F) Osteosarcoma
13. A 72-year-old woman comes to the physician because of a 6-month history of left knee pain.
The pain increases when she walks up or down stairs. She rates her pain as a 3 on a 10-point
scale. She has not had fever or morning stiffness. She has no history of serious illness and
takes no
medications. She is 160 cm (5 ft 3 in) tall and weighs 89 kg (196 lb); BMI is 35 kg/m2 Her
temperature is 37°C (98.6°F), pulse is 68/min, respirations are 12/min, and blood pressure is
128/70 mm Hg. Examination of the left knee shows crepitus on passive flexion and extension,
there is no
synovitis. An x-ray of the left knee is shown. Which of the following is the most appropriate initial
pharmacotherapy?
OA) Acetaminophen
B) Hydroxychloroquine
C) Ketorolac
D) Oxycodone
E) Prednisone
14. A 28-year-old primigravid woman at 7 weeks' gestation is admitted to the hospital because
of a 2-day history of shortness of breath, palpitations, and light-headedness. Pregnancy had
been uncomplicated. She has no history of serious illness, and her only medication is a prenatal
vitamin. Two months ago, she visited her
parents' farm in Wisconsin. The patient's temperature is 37.8°C (100°F), pulse is 38/min,
respirations are 20/min, and blood pressure is 110/70 mm Hg. Pulse oximetry on room air
shows an oxygen saturation of 96%. Forehead movement is decreased. There is left facial
droop, and the patient is unable to close her left eye.
An ECG is shown. Which of the following is the most appropriate next step in management?
A) Administration of ceftriaxone
B) Administration of doxycycline
C) Administration of mifepristone
OD) Administration of prednisone
E) Placement of a pacemaker
15. A previously healthy 53-year-old woman is brought to the emergency department 2 hours
after the onset of severe pressure and pain in her chest radiating down her left arm. She says
she fell off her bicycle last week and bruised the left side of her rib cage. She has been taking
naproxen for pain since then. She takes no
other medications. On arrival, she is in severe distress. Her pulse is 110/min, respirations are
25/min, and blood pressure is 80/50 mm Hg. Pulse oximetry on 2 L/min of oxygen by nasal
cannula shows an oxygen saturation of 85%. Examination shows cool, mottled skin. Jugular
venous pulsations are present 8 cm above
the sternal angle. Examination of the chest shows healing ecchymoses and mild tenderness
over the ribs on the left. Crackles are heard at both lung bases. S, and S, are normal; an S, is
heard. An ECG shows ST-segment depression and a left bundle branch block. Which of the
following is the most likely diagnosis?
OA) Acute pericarditis
B) Cardiac tamponade
OC) Myocardial infarction
OD) Pneumothorax
OE) Pulmonary embolism
16. A 23-year-old man comes to the physician for an examination prior to employment. He feels
well. He has no history of serious illness and takes no medications. He is in his first year of law
school and sleeps 4 to 5 hours nightly; he reports feeling stressed frequently because of the
demanding course load. He has smoked
one pack of cigarettes daily for 2 years and drinks one beer daily. He exercises by running for
30 minutes three times weekly. He is 178 cm (5 ft 10 in) tall and weighs 91 kg (200 lb); BMI is 29
kg/m2 His pulse is 80/min, and blood pressure is 132/96 mm Hg. Serum electrolyte
concentrations are within the reference ranges.
Urinalysis shows no abnormalities. In addition to advising smoking cessation, which of the
following is the most appropriate recommendation to manage this patient's increased blood
pressure?
A) Decreased alcohol consumption
OB) Increased amount of sleep
OC) Weight loss
D) Biofeedback
E) Hydrochlorothiazide therapy
17. Five days after admission to the hospital because of inoperable descending thoracic aortic
aneurysm with dissection, a 73-year-old man has moderate back pain. He has been treated with
acetaminophen-oxycodone and rest. He has hypertension, type 2 diabetes mellitus, and
hypercholesterolemia. On admission, his
medications were clopidogrel, levalbuterol, pravastatin, glipizide, metoprolol, lisinopril, and
aspirin. The clopidogrel and aspirin were discontinued on hospital day 1. Five years ago,
drug-eluting stents were placed in the left main coronary artery and circumflex artery for an
anterior myocardial infarction. His current
medications are nitroprusside, metoprolol, furosemide, acetaminophen-oxycodone, and
levalbuterol. He has smoked one pack of cigarettes daily for 50 years. His temperature is
38.3°C (101°), pulse is 110/min, and blood pressure is 180/106 mm Hg. Examination shows
livedo reticularis over the dorsum of the left lower
leg. There are petechiae over the dorsa of the feet and several small, purplish black lesions
over the left first, second, and third toes and the right second and third toes. The toes are tender
to palpation. There is lower abdominal tenderness; bowel sounds are increased. Laboratory
studies show:
Leukocyte count
14.000/mm3
Segmented neutrophils 80%
Eosinophils
13%
Lymphocytes
5%
Monocytes
2%
Erythrocyte sedimentation rate 130 mm/h
Serum creatinine
2.4 mg/dL
Urine
Protein
2+
RBC
20-30/hpf
Casts
multiple RBC and granular
Maltese crosses are noted. Which of the following is the most likely diagnosis?
OA) Antiphospholipid antibody syndrome
B) Bacterial endocarditis
C) Leukocytoclastic vasculitis
OD) Multiple cholesterol emboli syndrome
E) Systemic necrotizing vasculitis
18. A 13-year-old boy is brought to the emergency department by his father because of a 1-day
history of severe, left-sided throat pain. The pain is constant and increases when he swallows or
opens his mouth. He also has had mild shortness of breath during this time. He had an episode
of tonsillitis 5 years ago that resolved
with amoxicillin therapy. He has no other history of serious illness and takes no medications. His
temperature is 38.4°C (101.1°F), pulse is 100/min, respirations are 18/min, and blood pressure
is 110/65 mm Hg. Pulse oximetry on room air shows an oxygen saturation of 96%. Examination
shows edema of the left tonsil and
deviation of the uvula with almost complete occlusion of the pharyngeal space. The tonsil is
erythematous and fluctuant, there is a diffuse yellow exudate. Which of the following is the most
appropriate next step in management?
A) Administration of amoxicillin clavulanate
B) Administration of hydrocortisone
C) CT scan of the neck with contrast
D) Needle aspiration of the peritonsillar space
E) Tonsillectomy
19. A 47-year-old woman comes to the physician because of a 3-month history of a
nonproductive cough that is worse on weekdays when she is at work and improves during the
weekend. She reports that the cough resolves completely by the end of the weekend. She has
not had fever or weight loss. She does not smoke or
use illicit drugs. Four months ago, she began working at a dry cleaning business. She is in no
distress. Vital signs are within normal limits. An occasional end-expiratory wheeze is heard at
both lung bases with mild hyperresonance to percussion. The remainder of the examination
shows no abnormalities. Pulmonary
function tests are most likely to show which of the following findings?
FEV:FVC Ratio
Peak Expiratory Flow
Rate
Ο Α)
Decreased
decreased
B)
Decreased
increased
C)
Decreased
normal
Increased
decreased
Increased
increased
F
Increased
normal
G
Normal
decreased
H)
Normal
increased
1)
Normal
normal
20. A 62-year-old man comes to the physician because of a 1-year history of increasing
shortness of breath on exertion. He now has difficulty finishing yard work and climbing stairs in
his home. He has not had cough, fever, or night sweats. He has hypertension treated with
atenolol. He smoked two packs of cigarettes daily for
30 years but quit 12 years ago. He worked in the auto parts department of a local car dealership
until he had to retire last month because of his respiratory symptoms. His temperature is 36.2°C
(97.2°F), pulse is 88/min, respirations are 20/min, and blood pressure is 140/82 mm Hg. Pulse
oximetry on room air shows an
oxygen saturation of 90% at rest and 82% after walking 20 feet. Fine inspiratory crackles are
heard at both lung bases. The remainder of the examination shows no abnormalities. A
high-resolution CT scan of the chest shows a honeycomb appearance of the lungs. Which of the
following is the most likely diagnosis?
A) Asbestosis
B) Bronchiolitis obliterans
OC) Idiopathic pulmonary fibrosis
D) Mesothelioma
E) Pulmonary embolism
OF) Pulmonary Langerhans cell histiocytosis
21. A 12-month-old boy is brought to the physician 24 hours after the sudden onset of diarrhea.
His parents report that he has had approximately 10 episodes of watery stools with no blood or
mucus. He has vomited once. He has been drinking fluids and eating crackers but has not been
as active as usual. He appears alert
and is not in distress. His temperature is 38.3°C (101°F), pulse is 120/min, respirations are
26/min, and blood pressure is 90/60 mm Hg. Physical examination shows moist mucous
membranes. The oropharynx appears normal. The lungs are clear to auscultation. The
abdomen is soft and nontender with no distention. Bowel
sounds are normal. All extremities are warm and well perfused. Laboratory studies show:
Serum
Na+
139 mEq/L
K: 4.0 mEq/L
CI-
110 mEq/L
HCO3-
15 mEq/L
Urea nitrogen 10 mg/dL
Creatinine 0.5 mg/dL
Fecal leukocytes
none
Which of the following is the most appropriate next step in management?
OA) Oral rehydration
B) Oral bismuth subsalicylate therapy
C) Oral metoclopramide therapy
D) Oral penicillin therapy
OE) Intravenous administration of 0.9% saline
22. A 57-year-old man comes to the physician 3 days after undergoing colonoscopy screening
that showed an ulcerated mass of invasive, poorly differentiated adenocarcinoma in the mid
ascending colon. He feels well and has not had abdominal discomfort, black, tarry, or bloody
stools; or weight loss. He has sciatica treated
with ibuprofen as needed. Vital signs are within normal limits. Examination shows no
abnormalities. Results of laboratory studies are within the reference ranges. Which of the
following is the most appropriate next step in diagnosis?
OA) Abdominal ultrasonography
OB) CT scan of the abdomen
C) Bone scan
D) PET scan
E) Surgical exploration
23. An 84-year-old woman comes to the office because of a 1-week history of increasingly
severe pain and swelling of the left side of her face. The pain worsens with eating. She has mild
dementia, Alzheimer type, and hypertension. Her medications are donepezil and
hydrochlorothiazide. Her temperature is 38.1°C (100.5°F),
pulse is 86/min, and blood pressure is 144/90 mm Hg. Examination of the face shows left-sided
edema that is tender to palpation. On palpation of the left parotid gland, purulent discharge is
expressed from the orifice of the Stensen duct. Several tender nodes are palpated in the left
anterior cervical chain. Cardiopulmonary
examination shows no abnormalities. Which of the following is the most likely infectious agent?
OA) Clostridium perfringens
B) Moraxella catarrhalis
C) Pseudomonas aeruginosa
D) Staphylococcus aureus
E) Streptococcus pneumoniae
24. An 82-year-old woman, who resides in a skilled nursing care facility, is brought to the
emergency department because of a 2-day history of diffuse, severe abdominal pain, which has
worsened during the past day. She rates her pain as an 8 on a 10-point scale. Her last bowel
movement was 3 days ago. She has severe
diffuse arthritis, osteoporosis, hypertension, and scoliosis. Her medications are oxycodone,
alendronate, hydrochlorothiazide, and calcium and vitamin D supplementation. Her blood
pressure is 148/62 mm Hg. The abdomen is tympanitic to percussion with marked distention
and diffuse tenderness, there is no guarding or
rigidity. High-pitched bowel sounds are heard. X-rays of the abdomen are most likely to show
which of the following findings?
OA) Dilation of the sigmoid colon
B) Inflammatory mass and fecalith in the cecum
OC) Mucosal edema of the small intestine
D) Normal gas pattern in the large and small intestines
E) Presence of free air under the diaphragm
25. A 57-year-old woman with diabetic nephropathy comes to the physician for a routine
examination. She has chronic kidney disease, type 2 diabetes mellitus, and hypertension. Her
medications are glyburide, enalapril, and metoprolol. Her blood pressure is 128/90 mm Hg.
Cardiopulmonary examination shows no
abnormalities. There is trace pedal edema. A 24-hour urine collection shows a total protein
concentration of 5000 mg and creatinine clearance of 40 mL/min. Measurement of which of the
following is the most appropriate next step to prevent complications of this patient's kidney
disease?
OA) Hemoglobin concentration
B) Platelet count
C) Serum C-reactive protein concentration
D) Serum ferritin concentration
E) Serum thyroid-stimulating hormone concentration
26. A previously healthy 42-year-old woman, gravida 3, para 3, comes to the physician because
of a 3-month history of progressive urinary incontinence. She has loss of urine with coughing,
sneezing, and exercising. Her last bowel movement was 3 days ago. She has no history of
gynecologic illness. She has
gastroesophageal reflux disease treated with omeprazole and hypertension treated with
hydrochlorothiazide. Her children were born following uncomplicated vaginal deliveries. Her
blood pressure is 120/70 mm Hg. Physical examination shows no abnormalities. Pelvic
examination shows bulging of the anterior vaginal wall.
Her postvoid residual volume is 50 mL. Urinalysis shows no blood, WBCs, or nitrites. There is
loss furine on cough stress test. Which of the following is the most likely cause of this patient's
loss of urine?
A) Bladder tumor
B) Constipation
C) Hydrochlorothiazide therapy
D) Overflow incontinence
E) Urethral hypermobility
F) Urinary tract infection
27. A 32-year-old woman, gravida 2, para 1, at 38 weeks' gestation is admitted to the hospital in
labor. Her pregnancy has been complicated by the diagnosis of HIV infection. Antiretroviral
therapy with zidovudine (AZT), lamivudine (3TC), and nelfinavir was begun at 12 weeks'
gestation. She has tolerated therapy well. Her
most recent CD4+ T-lymphocyte count was 600/mm3 (Normal 500), and plasma HIV viral load
was 480 copies/mL. She wishes to attempt vaginal delivery. Which of the following is
contraindicated with this patient's pregnancy?
A) Amnioinfusion
B) Bulb suctioning of the newborn
C) Determination of umbilical artery pH at delivery
OD) Fetal scalp electrode placement
E) Intrauterine pressure catheter placement
28. A 22-year-old primigravid woman at 38 weeks' gestation comes to the physician as a new
patient because she recently moved to this area. Pregnancy has been uncomplicated. Her only
medication is a prenatal vitamin. Her father has factor VII deficiency. The patient is concerned
that her child may have hemophilia and
bleeding complications at the time of delivery. Which of the following is the most appropriate
physician recommendation?
A) Measurement of factor VIII concentration in the newborn at least 24 hours after delivery
B) Prophylactic maternal cryoprecipitate transfusion prior to delivery
C) Prophylactic maternal platelet transfusion prior to delivery
D) Scheduled cesarean delivery
OE) Testing of cord blood for factor VIII concentration after delivery if the baby is a boy
29. A 36-year-old woman comes to the clinic for a routine examination. She has hypertension
treated with lisinopril. She has no other history of serious illness or operative procedures. She
uses a copper IUD for contraception. She does not smoke cigarettes, drink alcohol, or use illicit
drugs. Her maternal aunt was diagnosed
with ovarian cancer at age 42 years. Her mother died in a motor vehicle collision at age 45
years. The patient's vital signs are within normal limits. On examination, a 2-cm, fixed nontender
mass is palpated in the upper outer quadrant of the right breast. Examination of the left breast
shows no abnormalities. Findings on fine-
needle biopsy of the mass are consistent with ductal carcinoma. Which of the following is the
most appropriate next step in management?
A) Bilateral mastectomy
B) Bilateral salpingo-oophorectomy
C) Genetic counseling for BRCA mutations
D) Pelvic ultrasonography and serum measurement of CA 125 concentration
E) Removal of the IUD
30. A 27-year-old woman is evaluated in the hospital 36 hours after undergoing cesarean
delivery at term because of a prolonged first stage of labor and rupture of membranes. She
currently reports abdominal pain and breast tenderness. Current medications are oxycodone
and acetaminophen. Her temperature is 38.9°C
(102°F), pulse is 100/min, respirations are 18/min, and blood pressure is 110/60 mm Hg.
Examination shows mild breast tenderness and induration. The incision site is clean and not
erythematous. The uterus is diffusely tender to mild palpation. A urine sample shows multiple
RBCs. Which of the following is the most
appropriate next step in management?
OA) Observation only
B) Breast binding
OC) Incentive spirometry
D) Endometrial culture
O E) Antibiotic therapy
31. A 51-year-old woman, gravida 1, para 1, comes to the office for an annual health
maintenance examination. She says she has been generally healthy and feels well but is
concerned about her risk for ovarian cancer because her friend of the same age recently was
diagnosed with such cancer. Medical history is significant
for an uncomplicated pregnancy and vaginal delivery. She breast-fed her child until he was age
6 months. Her only medications are a calcium supplement and daily multivitamin. She took an
oral contraceptive for 15 years and discontinued the therapy 1 year ago; her menses did not
resume after discontinuing the therapy.
She has not taken hormone replacement therapy. She exercises regularly and does not smoke
cigarettes. She has no known family history of cancer. All Pap smears in the past have shown
no abnormalities. She is 173 cm (5 ft 8 in) tall and weighs 63 kg (139 lb); BMI is 21 kg/m2 Vital
signs are normal. Physical
examination, including pelvic examination, discloses no abnormalities. Which of the following
factors in the patient's history is most strongly associated with a decreased risk for ovarian
cancer?
A) Absence of family history of ovarian cancer
B) BMI
OC) Breast-feeding history
D) Negative smoking history
E) Oral contraceptive use
F) Postmenopausal status
32. A 17-year-old boy is brought to the physician for an examination prior to participation in
sports. He has no history of serious illness and takes no medications. He was diagnosed with
intellectual developmental disorder at the age of 5 years. He is 165 cm (5 ft 5 in) tall and weighs
50 kg (110 lb); BMI is 18 kg/m2 Vital signs
are within normal limits. Examination shows minimal facial hair. There is mild bilateral
gynecomastia. The testes are Tanner stage 1. Which of the following is the most likely
explanation for this patient's gynecomastia?
OA) Congenital adrenal hyperplasia
B) Constitutional delay of puberty
C) Incomplete androgen insensitivity syndrome
OD) Seminiferous tubule dysgenesis (Klinefelter) syndrome
E) Normal variant of puberty
33. A 16-year-old boy with type 1 diabetes mellitus is brought to the emergency department by
his parents because of a 1-hour history of vomiting. The parents say he was at an overnight
camp during the past week. When he returned 12 hours ago, he was not as active as usual and
said he was tired. The patient told his
parents that he had taken his insulin while at camp, but when they checked his bag, his supply
of insulin indicated that he had not been taking it regularly. On arrival, he appears ill and drowsy
but fully oriented. There is a sweet odor to his breath. His temperature is 37°C (98.6°F), pulse is
120/min, respirations are 26/min
and deep, and blood pressure is 115/60 mm Hg. Examination shows dry mucous membranes.
No other abnormalities are noted. Laboratory studies show:
Fingerstick blood glucose
>600 mg/dL
Serum HCO3-
10 mEq/L
Arterial blood gas analysis on room air shows:
pH
PCO2
7.13
32 mm Hg
95 mm Hg
PO2
Administration of which of the following is the most appropriate initial step in management?
OA) Intramuscular promethazine
B) Intravenous bicarbonate
C) Intravenous insulin
OD) Intravenous 0.9% saline bolus
E) Subcutaneous insulin
OF) Sublingual ondansetron
34. A 32-year-old woman comes to the physician because of a 2-week history of double vision
and dry, red eyes. She also has had a 4.5-kg (10-b) weight loss and increased fatigability during
this time. Her only medication is an oral contraceptive. She is 163 cm (5 ft 4 in) tall and weighs
50 kg (110 lb); BMI is 19 kg/m2 Her
temperature is 37.2°C (99°F), pulse is 100/min, respirations are 16/min, and blood pressure is
135/60 mm Hg. Examination shows prominent-appearing eyes and bilateral scleral injection.
She has limited upward gaze, and she reports severe diplopia when she looks up; the eyes
open wider on downward gaze. Visual
acuity is 20/40 in the left eye and 20/30 in the right. Which of the following is the most
appropriate next step in diagnosis?
OA) Schirmer test
B) Acetylcholine receptor antibody titers
C) Measurement of serum angiotensin-converting enzyme activity
D) Measurement of serum glucose concentration
OE) Thyroid function testing
35. A 42-year-old woman is brought to the emergency department 30 minutes after her husband
found her unconscious in her bed. She has a 6-year history of systemic lupus erythematosus
treated with prednisone. Her husband says she has been on a drinking binge for the past 7 days
and has not been eating. Three days
ago, she had the onset of rhinitis, conjunctivitis, and a nonproductive cough. She has a
long-standing history of alcoholism. She moans in response to painful stimuli. Her temperature
is 38.9°C (102°F), pulse is 110/min, respirations are 22/min, and palpable systolic blood
pressure is 80 mm Hg. The lungs are clear to
auscultation and percussion. Abdominal examination shows no abnormalities. Fluid
resuscitation is begun. Which of the following is the most appropriate initial step in
management?
A) Serum antinuclear antibody assay
OB) Corticosteroid therapy
C) Cyclosporine therapy
D) Ganciclovir therapy
E) Nonsteroidal anti-inflammatory drug therapy
36. A 3-year-old girl is brought to the emergency department after her mother found her in the
kitchen next to an open bottle of drain cleaner that had been stored under the sink. The bottle
was still almost full, but some of its contents were spilled on the patient's clothes and on the
floor. On arrival, she is alert, crying, and
drooling but in no respiratory distress. Her pulse is 120/min, respirations are 30/min, and blood
pressure is 95/70 mm Hg. The mouth and oral mucosa is bright red, but there are no ulcers in
the oropharynx. The lungs are clear to auscultation. Abdominal examination shows no
abnormalities. The extremities are well perfused.
An x-ray of the chest shows clear lung fields and normal cardiac silhouette and mediastinum.
Which of the following is the most appropriate next step in management?
A) Barium swallow
OB) Administration of activated charcoal
C) Administration of ipecac
D) Esophagoscopy
OE) Gastric lavage by nasogastric tube
37. A 9-year-old boy is brought to the office by his parents because of bilateral groin hernias
discovered by a nurse on examination 5 days ago at an urgent care clinic. The patient's parents
are migrant agricultural workers, and the patient's medical care has been inconsistent and
infrequent. The parents say their son has no
history of serious illness, he has only occasional upper respiratory tract infections that resolve
after treatment with over-the-counter medications. The patient currently receives no
medications. The nurse reports that she administered several routine vaccines to the patient
because he had not received them yet. He is at the
80th percentile for height and 20th percentile for weight. Examination shows an elongated face,
droopy eyelids, and mild scoliosis. There is mild pectus carinatum. Cardiopulmonary and
abdominal examinations show no abnormalities. The distance encompassed by the patient's
outstretched upper extremities is 1.2 times
his height. On Valsalva maneuver, bilateral hernias below the inguinal ligaments are easily
reducible. In addition to scheduling operative repair of the hernias, which of the following is the
most appropriate next step in management?
OA) Echocardiography
B) Karyotype analysis
C) MRI of the spine
D) Pulmonary function tests
E) Tonometry
38. A prospective cohort study is conducted to assess the likelihood of undergoing repeat
cesarean delivery after a trial of labor with or without oxytocin. Oxytocin use and mode of
delivery are recorded from hospital records for all deliveries in one calendar year. The results
show:
Repeat Cesarean Delivery
Yes No Total
Yes
Use of Oxytocin
160 840 1000
No 320 680 1000
Total 480 1520 2000
Based on these results, which of the following represents the relative risk for a repeat cesarean
delivery with oxytocin use?
A) 0.16
OB) 0.33
OC) 0.50
D) 0.67
E) 0.84
39. A study is conducted to assess the efficacy of a new drug for treatment of pruritus
secondary to primary sclerosing cholangitis (PSC). A total of 300 patients with PSC are enrolled
and randomly assigned to one of two groups: 150 patients receive the new drug (Drug X), and
150 patients receive the placebo. Pruritus is
evaluated at the start of the study and after 1 week of treatment. The results show:
Number of Patients
Group
Pruritus at 1 Week
Adverse Events
Drug X
40
25
Placebo
90
18
Based on these data, which of the following represents the number needed to treat for 1 week
with Drug X to prevent one case of PSC pruritus in this population?
A) 3
B) 7
OC) 40
OD) 50
OE) Cannot be determined from the data provided
40. State health officials are planning to implement a new skin cancer screening program in
agricultural workers in several southern states who are occupationally exposed to the sun. The
screening program will consist of annual visual skin examinations. Some researchers are
concerned about the limited effectiveness of the
program. Which of the following is most likely to cause overestimation of the effectiveness of
this screening program?
OA) Confounding by geographic location
B) Healthy worker effect
OC) Length bias
D) Random error
41. A study is conducted to assess the efficacy of a new drug for the treatment of hypertension.
A total of 6000 patients are enrolled in the study; 50% of the patients receive the new drug and
50% receive standard antihypertensive therapy. The goal is to confirm efficacy and assess for
adverse effects. The study builds on other
studies done previously to assess safe dosing range, drug administration protocol, and initial
efficacy. Which of the following best describes the phase of this clinical trial?
A) O
B) 1
O C)2
D)3
E) 4
42. A 67-year-old man comes to the physician because of easy fatigability and generalized
weakness for 3 months and left chest pain for 1 month. The chest pain is worse on deep
inspiration. He appears slightly pale. There is tenderness over the left 8th and 9th ribs laterally.
Examination shows no other abnormalities. His
hematocrit is 28%. Serum and urine protein electrophoresis shows a monoclonal spike. A
biopsy specimen of bone marrow shows greater than 50% plasma cells. An X-ray of the chest
shows 1-to 1.5-cm areas of radiolucency in both ribs corresponding to the sites of tenderness.
This patient's condition makes him most
susceptible to infection with which of the following organisms?
A) Aspergillus fumigatus
B) Escherichia coli
C) Herpes zoster virus
D) Mycobacterium tuberculosis
OE) Pneumocystis jirovecii
F) Streptococcus pneumoniae
44. A 23-year-old man with schizophrenia is brought to the emergency department by his
mother 28 hours after the onset of progressive confusion and agitation. He has had increased
thirst during this period. One week ago, he was discharged after treatment for the second
exacerbation of schizophrenia. Current medications
include haloperidol and benztropine. He appears flushed. His temperature is 38°C (100.4°F),
pulse is 100/min, and blood pressure is 160/90 mm Hg. Examination shows dry mucous
membranes. Pupils are 4 mm and reactive to light. On mental status examination, he is not
oriented to person, place, or time. Which of the
following is the most likely explanation for these findings?
O A) Anticholinergic delirium
B) Exacerbation of schizophrenia
C) Neuroleptic malignant syndrome
D) Oxycodone overdose
E) PCP (phencyclidine) intoxication
45. A 24-year-old man comes to the clinic because of a 2-day history of severe mouth pain,
fever, sore throat, and malaise. He has no known sick contacts. He has no history of serious
illness and takes no medications. He recently returned home from active duty deployment in the
US Air Force. His temperature is
38.7°C (101.7°F). Examination shows cervical lymphadenopathy. There are multiple small
vesicles over the inside of the mouth and posterior oropharynx. Soft tissue fullness is noted.
Which of the following is the most appropriate next step in diagnosis?
A) Biopsy of an oral vesicle
B) Monospot test
OC) Polymerase chain reaction test of an open vesicle
D) Swab and culture of the posterior oropharynx
E) Wet mount preparation of oral cavity scrapings with KOH
46. A 4-year-old girl is brought to the physician because of a 3-month history of episodic
abdominal pain and loose stools that recur several times weekly. She has not had nausea,
vomiting, or decreased appetite. Her development has been appropriate for age. She received
the diagnosis of type 1 diabetes mellitus 9 months
ago, and her most recent hemoglobin At was 7.5%. Her only medication is insulin. She appears
well. She is at the 25th percentile for height and 10th percentile for weight, as she was at her
last examination 4 months ago. Her temperature is 36.7°C (98°F), pulse is 100/min, respirations
are 24/min, and blood pressure is
95/60 mm Hg. The conjunctivae are pale. A grade 1/6, systolic ejection murmur is heard at the
left sternal border. The abdomen is mildly distended but soft and nontender, there is no
hepatosplenomegaly. The remainder of the examination shows no abnormalities. Laboratory
studies show:
Hemoglobin
10 g/dL
Mean corpuscular volume 67 um (N=70-74)
Leukocyte count
8500/mm3
Platelet count
250,000/mm3
Red cell distribution width 18% (N=13%-15%)
Additional serum studies are most likely to show an increase in which of the following?
A) 01-Antitrypsin concentration
B) Ferritin concentration
C) Folic acid concentration
D) Tissue transglutaminase IgA antibodies
E) Total bilirubin concentration
47. A 42-year-old woman, gravida 5, para 4, at 14 weeks' gestation comes to the physician for a
routine prenatal visit. Prior to conception, menses had occurred at regular 28-day intervals. Her
temperature is 37°C (98.6°F), pulse is 80/min, respirations are 20/min, and blood pressure is
120/80 mm Hg. Examination shows a
closed cervix and a uterus consistent in size with a 6-week gestation. Ultrasonography shows a
normal uterus with a gestational sac and no cardiac activity. Which of the following is the most
likely diagnosis?
OA) Blighted ovum
B) Ectopic pregnancy
C) Gestational trophoblastic disease
D) Pituitary adenoma
E) Septic abortion
F) Uterine didelphys
48. A 77-year-old man is brought to the physician by his wife because of a 1-year history of poor
balance, weakness of his legs, and numbness of his toes and feet. His wife states that he is not
as mentally sharp as he was 1 year ago. He has peptic ulcer disease and hypertension. Current
medications are hydrochlorothiazide
and ranitidine. His blood pressure is 148/88 mm Hg. Examination shows normal muscle tone
and strength in the upper and lower extremities. Deep tendon reflexes are 2+ in the upper
extremities, 3+ at the knees, and absent at the ankles. Babinski sign is present. Proprioception
and sensation to vibration are markedly
decreased at the ankles and toes. He has a broad-based but steady gait. On mental status
examination, he is oriented to month, day, and year. He can name the current president but
none of the previous presidents. Mean corpuscular volume is 105 um and serum lactate
dehydrogenase activity is 210 U/L. A serologic test
for syphilis is negative. MRI of the brain shows mild cortical atrophy, enlarged ventricles, and
multiple small areas of T2 hyperintensities in the white matter. Which of the following is the most
likely diagnosis?
A) Amyotrophic lateral sclerosis
B) Dementia, Alzheimer type
OC) Multi-infarct (vascular) dementia
D) Normal-pressure hydrocephalus
O E) Vitamin B12 (cobalamin) deficiency
49. A 17-year-old boy with intellectual developmental disorder is brought to the physician
because of low-grade fever and abdominal pain for 6 days. His temperature is 37.8°C (100.1°F),
pulse is 110/min, respirations are 22/min, and blood pressure is 120/70 mm Hg. Examination
shows splinter hemorrhages under the nails. A
grade 2/6 systolic murmur is heard best at the upper left sternal border. There is a systolic
ejection click. S, and S, are normal. Abdominal examination shows splenomegaly. Laboratory
studies show:
Hemoglobin
9.1 g/dL
Leukocyte count
30,000/mm3
Platelet count
928,000/mm3
Erythrocyte sedimentation rate 110 mm/h
Urine
Blood
2+
Protein
Which of the following is the most appropriate next step in management?
A) 24-Hour urine collection for measurement of protein and creatinine concentrations
OB) Blood cultures
C) Ultrasonography of the abdomen
D) CT scan of the abdomen
E) Broad-spectrum antibiotic therapy
50. A 42-year-old man is admitted to the hospital because of a 2-day history of fever, chills,
cough, and shortness of breath. He currently takes atenolol for hypertension and pravastatin for
hyperlipidemia. He has smoked one pack of cigarettes daily for 20 years. He underwent
splenectomy at the age of 18 years for injuries
sustained in a motorcycle collision. He appears pale and is in moderate distress. His
temperature is 40°C (104°F), pulse is 120/min, respirations are 22/min, and blood pressure is
100/60 mm Hg. Crackles and decreased breath sounds are heard over the left lower lung field
there is dullness to percussion. Examination
shows no other abnormalities. A Gram stain of sputum shows gram-positive diplococci.
Treatment with ceftriaxone is begun. An x-ray of the chest shows an infiltrate and pleural
effusion on the left. Thoracentesis is performed. Laboratory studies show:
Pleural fluid
pH
6.8
Glucose
30 mg/dL
Lactate dehydrogenase
850 U/L
Arterial blood gas analysis on room air.
pH
Pco2
PO2
7.44
32 mm Hg
68 mm Hg
A Gram stain of the pleural fluid shows many segmented neutrophils and gram-positive
diplococci. Which of the following is the most appropriate next step in management of the
pleural effusion?
A) Broaden antibiotic coverage for anaerobic bacteria
B) Broaden antibiotic coverage for mycobacteria
OC) Placement of a left chest tube
D) Surgical debridement of the pleural space
E) No specific therapy for the effusion indicated
2. A 10-year-old boy is brought to the office by his mother because of a 7-day history of fever
and nasal congestion. He has allergic rhinitis. He has had multiple episodes of otitis media and
sinusitis and two episodes of pneumonia. His only routine medication is cetirizine. He appears ill
but is in no distress. He is at the
30th percentile for height, 40th percentile for weight, and 35th percentile for BMI. His
temperature is 37.8°C (100.0°F), pulse is 80/min, respirations are 14/min, and blood pressure is
110/70 mm Hg. Examination discloses an erythematous posterior pharynx and green nasal
secretions. The left tympanic membrane is bulging
and immobile with insufflation. No other abnormalities are noted. This patient most likely has a
deficiency of which of the following immunoglobulins (Ig)?
O A) IgA
B) IgD
C) IgE
OD) IgG
OE) IgM
3. A 67-year-old woman is brought to the emergency department by her husband after having a
generalized tonic-clonic seizure at home. Her husband reports that shortly after awakening this
morning, she had involuntary twitching of the right side of her face and right arm, lost
consciousness, stiffened, and had generalized
shaking for 2 minutes. She has type 2 diabetes mellitus and hypertension. Four months ago,
she received the diagnosis of adenocarcinoma of the breast, metastatic to the lungs and bone.
Current medications include metformin, atenolol, and letrozole. She is drowsy but able to follow
simple commands with all extremities. Her
temperature is 37°C (98.6°F), pulse is 90/min, and blood pressure is 160/90 mm Hg. She has a
right lower facial droop. There is no meningismus. The pupils are 3 mm and reactive to light.
The fundi are normal. There is conjugate gaze deviation to the patient's left. Muscle strength is
4/5 in the right upper extremity. Deep
tendon reflexes are 3+ in the right upper extremity and 2+ elsewhere. Babinski sign is absent
bilaterally. Laboratory studies show:
Hemoglobin 12 g/dL
Leukocyte count 3000/mm3
Platelet count
80.000/mm3
Serum
Na+
128 mEq/L
Glucose
70 mg/dL
Which of the following is the most likely cause of this patient's seizure?
A) Brain metastasis
B) Hypoglycemia
C) Hyponatremia
OD) Intracerebral hemorrhage
E) Paraneoplastic limbic encephalitis
4. A 52-year-old man comes to the physician because of a 2-year history of progressive hearing
loss in his right ear. He says he initially only noticed gradual changes in his hearing but has had
severe hearing loss during the past 6 months. He also has had ringing in his right ear for 12
months and episodes of light-headedness
and numbness and tingling on the right side of his face during the past month. He has a history
of mild hypertension. Current medications include hydrochlorothiazide and 81-mg aspirin. He
has been employed operating heavy equipment on construction sites for 30 years. He is not in
distress. Vital signs are within normal
limits. The external ear canals and tympanic membranes appear normal. He cannot hear
whispered voice in the right ear. Air conduction is greater than bone conduction on the right. A
tuning fork held over the forehead at midline is heard better on the left. Results of laboratory
studies are within the reference range. Which of
the following is the most likely diagnosis?
A) Acoustic neuroma (vestibular schwannoma)
B) Benign positional vertigo
C) Drug toxicity
OD) Meniere disease
OE) Multiple sclerosis
F) Vertebrobasilar insufficiency
5. A 32-year-old man comes to the clinic because of a slow-healing lesion on his nose that has
been present for the past year. The patient is a member of the US National Guard and says he
first noticed the lesion 2 months after he returned home from deployment to Southwest Asia.
The lesion initially appeared as two small
pimples but gradually enlarged to form a single open sore, eventually developing a scaly crust.
The lesion has not been painful or itchy. Aside from several insect bites and itching at various
times during his deployment, he had no skin trauma or infections and did not have any
significant contact with the local population or
animals. He received all required vaccines, including smallpox, anthrax, and typhoid, prior to his
deployment. He has no history of serious illness and takes no medications. Vital signs are within
normal limits. A photograph of the lesion is shown. The lesion is not tender. Which of the
following is the most likely diagnosis?
A) Anthrax
OB) Leishmaniasis
C) Psoriasis
D) Squamous cell carcinoma
E) Tinea corporis
7. A 42-year-old woman comes to the physician because of a 1-day history of severe pain,
swelling, and warmth in her left knee. She notes that she awoke with the pain 1 day after she
did an aerobic kickboxing workout. Her temperature is 37.2°C (99°F), pulse is 86/min, and blood
pressure is 120/70 mm Hg. The upper aspect
of the left knee is erythematous and warm to the touch with an effusion. Range of motion is
limited by pain. The remainder of the examination shows no abnormalities. Arthrocentesis is
performed. Joint fluid analysis shows a leukocyte count of 17,000/mm3 with 90% segmented
neutrophils and intracellular calcium
pyrophosphate crystals. Which of the following is the most appropriate pharmacotherapy at this
time?
A) Oral acetaminophen
B) Oral doxycycline
C) Oral hydroxychloroquine
D) Oral ibuprofen
E) Intravenous nafcillin
8. A 66-year-old man comes to the office because of worsening pain in his right shoulder that
began 1 day after installing overhead tile in his house 9 months ago. Initially, the pain only
affected his muscle strength but during the past month he has been unable to raise his arm
higher than 90 degrees, even when the pain is
mild. The pain is worse at night and improves moderately an hour or two after he wakes up in
the morning. Medical history is noncontributory; the patient takes ibuprofen for the shoulder
pain. Vital signs are within normal limits. Examination of the right shoulder shows mild muscular
atrophy with no tenderness to palpation.
Passive range of motion in the shoulder is full, except that the patient reports a "twinge" of
discomfort during testing. Drop-arm testing is positive. Which of the following is the most likely
diagnosis?
A) Adhesive capsulitis
B) Biceps tendinitis
C) Cervical nerve impingement
D) Labral tear
O E) Rotator cuff tear
9. A 23-year-old man comes to the emergency department 45 minutes after the sudden onset of
palpitations and mild shortness of breath that began while he was drinking beer with friends. He
has not had chest pain or loss of consciousness. He has no history of serious illness and takes
no medications. On arrival, he appears
anxious. He is alert and fully oriented. His pulse is 138/min and regular, and blood pressure is
100/65 mm Hg; other vital signs are within normal limits. Pulse oximetry on room air shows an
oxygen saturation of 99%. The patient's tachycardia ends abruptly during the examination. The
remainder of the examination shows no
abnormalities. Which of the following is the most likely diagnosis?
A) Atrial fibrillation
B) Atrioventricular nodal reentrant tachycardia
OC) Multifocal atrial tachycardia
D) Sinus tachycardia
OE) Ventricular tachycardia
F) Wandering atrial pacemaker
10. A previously healthy 32-year-old man comes to the office because of a 6-month history of
palpitations, fatigue, and shortness of breath on exertion. He also has had occasional chest pain
after eating large meals. He appears healthy and is not in acute distress. His pulse is 88/min,
respirations are 12/min, and blood pressure is 138/86 mm Hg.
Carotid pulses are forceful. Cardiac examination shows a left ventricular lift. An S. gallop and
grade 3/6, harsh, systolic crescendo-decrescendo murmur are heard best at the lower left
sternal border and apex. Results of Valsalva maneuver and ECG are shown. Which of the
following is the most likely diagnosis?
A) Aortic sclerosis
B) Aortic stenosis
OC) Hypertrophic obstructive cardiomyopathy
D) Mitral regurgitation
E) Mitral stenosis
11. A 57-year-old woman comes to the emergency department because of a 1-day history of
constant sharp chest pain and shortness of breath. The pain is located to the left of her sternum,
increases when she lies on her back, and decreases when she sits and
leans forward. She has had no cough, sputum production, recent trauma, or recent respiratory
tract infection. She has hypertension and reactive airway disease. Medications are inhaled
tiotropium, inhaled salmeterol-fluticasone, montelukast, amlodipine, and
81-mg aspirin. Her temperature is 37.6°C (99.6°F), pulse is 62/min, respirations are 24/min, and
blood pressure is 87/55 mm Hg. Pulse oximetry on room air shows an oxygen saturation of
88%. Jugular venous pressure is 9 cm H20. The remainder of the
examination shows no abnormalities. Her hematocrit is 39%, leukocyte count is 15,800/mm?,
and platelet count is 252,000/mm3 Serum electrolyte concentrations are within the reference
ranges. An ECG shows diffuse ST-segment elevation. A CT scan of the
chest with contrast is shown. Which of the following is the most appropriate next step in
diagnosis?
A) Measurement of peak expiratory flow rate
OB) Measurement of serum brain natriuretic peptide concentration
OC) Measurement of serum cortisol concentration
D) Serum D-dimer assay
E) Blood cultures
OF) Echocardiography
OG) Pulmonary angiography
Page: 41 of 67
Compressed 8:1
IM: 41 SE: 2
12. A 25-year-old man comes to the physician because of a 2-day history of right calf pain that
began after he played in a soccer match. The pain is worse with walking and resolves with rest
or acetaminophen therapy. He is otherwise healthy and takes no other medications. There is no
family history of serious illness. The
patient traveled outside the USA by plane 4 months ago. He drinks 48 oz of beer weekly. He
does not smoke cigarettes or use illicit drugs. His pulse is 85/min, respirations are 14/min, and
blood pressure is 120/80 mm Hg. Pulse oximetry on room air shows an oxygen saturation of
98%. Skin examination shows ecchymoses
over the anterior and lateral aspects of the right calf. There are no cords. The calves are
symmetric in size, and there is no warmth. The right lower extremity is diffusely tender to deep
palpation below the knee. Cardiopulmonary and neurologic examinations show no
abnormalities. Gait is normal. Serum D-dimer
concentration is less than 0.2 pg/mL (N<0.5) by ELISA. Which of the following is the most
appropriate next step in diagnosis?
A) Compression ultrasonography of the right lower extremity
B) CT pulmonary angiography
C) Measurement of prothrombin and partial thromboplastin times
D) MRI of the right lower extremity
E) Venography of the right lower extremity
OF) No further testing is indicated
13. Five days after discharge from the hospital following treatment of ischemic left hemispheric
cerebral infarction, an 82-year-old man is brought to the physician because of repeated
coughing after drinking liquids. Recovery in the hospital was complicated by right lower lobe
pneumonia that resolved after treatment with
antibiotics. The patient has lived in a skilled nursing care facility since discharge. He has no
known allergies. His medications are lisinopril and aspirin. He is currently not in respiratory
distress. Vital signs are within normal limits. Muscle strength is 375 in the right extremities and
5/5 in the left extremities. On mental status
examination, he is alert and fully oriented. His speech is dysarthric, and he has occasional
difficulty finding words. Which of the following is most likely to decrease this patient's risk for
pulmonary complications?
A) Addition of metoclopramide with meals
B) Addition of a thickening agent to all liquids
C) Direct supervision when drinking liquids
OD) Tracheostomy
OE) Use of a straw when drinking liquids
14. A 32-year-old woman with Crohn disease is being placed on total parenteral nutritional
support. During insertion of a central venous catheter into the left subclavian vein, she has the
sudden onset of severe chest pain, dyspnea, and agitation. She is supine, and a sterile drape is
placed over her. Her medications are
infliximab and prednisone. She appears malnourished and anxious. She is 157 cm (5 ft 2 in) tall
and weighs 45 kg (100 lb); BMI is 18 kg/
m2 Examination shows diaphoresis. Which of the following is the most appropriate next step in
management?
A) Leave the patient supine and reassure her that the procedure is going well
B) Remove the sterile drape and seat the patient in the upright position
C) Place the patient into Trendelenburg and left lateral decubitus position
D) Attempt to place the catheter in the right subclavian vein
E) Administer intravenous lorazepam and perform pericardiocentesis
15. An 82-year-old man comes to the physician for a follow-up examination 1 week after
discharge from the hospital, where he received treatment for streptococcal pneumonia. He
received a 7-day course of intravenous ceftriaxone in the hospital, then completed a 4-day
course of oral doxycycline at home. His fever, shortness
of breath, cough, and chills resolved by the time of hospital discharge. He has hypertension and
hyperlipidemia. His current medications are metoprolol
, lisinopril, and atorvastatin. He smoked one pack of cigarettes daily for 50 years but quit 10
years ago. His blood pressure is 128/84 mm Hg. Breath sounds are decreased
at the right lung base; no crackles, rhonchi, or rubs are heard. The remainder of the examination
shows no abnormalities. Laboratory studies show:
Hemoglobin 13.8 g/dL
Hematocrit
41%
Leukocyte count 5200/mm3
A sputum culture shows no leukocytes or organisms. A chest x-ray is shown. Which of the
following is the most appropriate next step in diagnosis?
OA) CT scan of the chest
B) Lung biopsy
C) Pleural biopsy
D) Repeat sputum culture
OE) No further testing is indicated
16. A 49-year-old man is admitted to the hospital because of a 2-year history of progressive,
oily, foul-smelling diarrhea and an unintentional 18-kg (40-Ib) weight loss. He has not had
nausea, vomiting, melena, or rectal bleeding. He also has a 5-year history of intermittent joint
pain and swelling of the wrists, hips, and knees.
His wife reports that his memory has declined during the past month. He has no other history of
serious illness and takes no medications. He is a farmer. He is 178 cm (5 ft 10 in) tall and
weighs 57 kg (125 lb); BMI is 18 kg/m2. His temperature is 36.3°C (97.3°F), pulse is 88/min,
respirations are 16/min, and blood pressure
is 124/82 mm Hg. Examination shows cervical, axillary, and inguinal lymphadenopathy. The
abdomen is nontender to palpation; no masses are palpated. There is no hepatosplenomegaly.
Bowel sounds are normal. Test of the stool for occult blood is positive. Endoscopic biopsy
specimen of the small intestine shows
numerous periodic acid-Schiff-positive macrophages in the lamina propria. Which of the
following is the most appropriate next step in management?
A) Ceftriaxone therapy followed by trimethoprim-sulfamethoxazole therapy
B) Omeprazole, amoxicillin, and clarithromycin therapy
O C) Pancreatic enzyme therapy
D) Prednisone therapy
E) Recommendation for gluten-free diet
17. A 47-year-old woman comes to the physician because of a 1-year history of difficulty
swallowing liquids and solids. She says she often feels food and drink stick in the middle of her
chest. For 8 weeks, she has been awakening at night after regurgitating partially digested food.
After meals, she has mild heartburn that is
occasionally associated with a mild sensation of fullness and discomfort in her chest. The
heartburn is not relieved by over-the-counter antacids. She has had a 3.2 kg (7-b) weight loss
during the past 6 months. She has no history of serious illness. She does not smoke cigarettes
or drink alcohol. She takes a daily
multivitamin. Examination shows no abnormalities. Which of the following is the most likely
diagnosis?
O A) Achalasia
B) Esophageal carcinoma
OC) Esophageal spasm
D) Gastroesophageal reflux disease
E) Pharyngoesophageal (Zenker) diverticulum
18. A 35-year-old woman comes to the office because of a 4-month history of progressive
fatigue, generalized itching, and dry eyes and mouth. She has no history of serious illness and
takes no medications. Her pulse is 80/min, and blood pressure is 123/77 mm Hg. Examination
shows excoriations over the extremities. The
liver span is 20 cm. The remainder of the examination shows no abnormalities. Laboratory
studies show:
Serum
Glucose
88 mg/dL
Creatinine
0.8 mg/dL
Total bilirubin
1.5 mg/dL
Alkaline phosphatase
300 U/L
AST
30 U/L
ALT
35 U/L
Antimitochondrial antibodies positive
Liver biopsy shows lymphocytic infiltrates in portal tracts with loss of medium-sized interlobar
bile ducts. Which of the following is the most appropriate pharmacotherapy?
OA) Azathioprine
B) Colchicine
C) Methotrexate
D) Prednisone
OE) Propranolol
OF) Ursodeoxycholic acid
20. A 16-day-old girl is brought to the physician for a well-child examination. She has a 1-week
history of increasingly yellow skin. She was born at term following an uncomplicated pregnancy
and delivery. Her birth weight was 3005 g (6 lb 10 oz). Newborn screening showed no
abnormalities. She receives no medications. She
is breast-feeding well. Today, she weighs 3062 g (6 lb 12 oz). Vital signs are within normal limits.
Examination shows generalized jaundice and scleral icterus. Laboratory studies show a serum
bilirubin concentration of 15 mg/dL, with a direct component of 10 mg/dL and an indirect
component of 5 mg/dL. Urinalysis shows
no reducing substances or pus. Which of the following is the most appropriate next step in
management?
A) Weekly monitoring of serum bilirubin concentration
OB) Abdominal ultrasonography
C) Upper gastrointestinal series
D) Oral administration of bile salts
E) Oral neomycin therapy
21. A previously healthy 27-year-old woman comes to the physician because of frequent
urination and dysuria for 2 days. She has had no fever, chills, or back pain. She has had two
similar episodes over the past 12 months treated with trimethoprim-sulfamethoxazole.
Examination shows suprapubic tenderness to palpation.
There is no costovertebral angle tenderness. Urinalysis shows 20-30 WBC/hpf and is positive
for nitrites. Which of the following is the most appropriate measure to prevent recurrent
symptoms?
A) Emptying the bladder one-half hour before vaginal intercourse
B) Ingestion of sodium bicarbonate in one-half glass of water 1 hour before vaginal intercourse
C) Amoxicillin therapy for the first 3 days of each month
OD) Postcoital oral trimethoprim-sulfamethoxazole therapy
E) Propantheline therapy three times daily at the onset of symptoms
22. An 18-year-old man who is a college student comes to the office for a follow-up examination
3 days after routine dipstick urinalysis during a sports physical examination showed proteinuria.
Repeat dipstick urinalysis 2 days later was negative for protein. He has no history of serious
illness and takes no medications. Today,
vital signs are within normal limits. Examination shows no abnormalities. Microscopic
examination of urine sediment shows no abnormalities. Which of the following is the most
appropriate next step in management regarding the change in urine findings?
A) Bone marrow biopsy
B) 24-Hour protein collection
C) Kidney biopsy
D) Urine protein immunoelectrophoresis
OE) No management is indicated
23. A 37-year-old woman, gravida 4, para 4, has continued to have vaginal bleeding 2 hours
after forceps delivery of a 4300-9 (9-b 8-oz) newborn at term. Pregnancy was complicated by
gestational diabetes. During delivery, shoulder dystocia required hyperflexion of the maternal
hips, suprapubic pressure, and manual rotation
of the fetal shoulders. The patient's pulse is 115/min, and blood pressure is 90/55 mm Hg. The
uterine fundus is firm and palpable at the umbilicus, and the placenta appears to have been
delivered intact. Which of the following is the most likely cause of the vaginal bleeding?
OA) Genital tract laceration
B) Myometrial atony
C) Uterine inversion
D) Uterine rupture
E) Normal lochia
24. A previously healthy 27-year-old woman, gravida 1, para 1, comes to the physician because
of a 2-week history of anxiety, irritability, and difficulty sleeping. During this time, she has noticed
a mild tremor of her hands. Two months ago, she gave birth to a healthy male newborn at term.
Delivery was uncomplicated. She is not
breast-feeding. She initially attributed her symptoms to giving birth. Her only medication is a
prenatal vitamin. She appears slightly anxious. Her temperature is 37.1°C (98.8°F), pulse is
108/min, respirations are 16/min, and blood pressure is 122/74 mm Hg. Examination shows a
normal-sized thyroid that is slightly tender to
palpation. There is a fine tremor of the outstretched hands. Laboratory studies show:
Hemoglobin
Hematocrit
35%
Mean corpuscular volume
82 um
Leukocyte count
9800/mm3
Platelet count
410,000/mm3
Serum
Thyroid-stimulating hormone
0.3 U/mL
Thyroxine (T)
12 ug/dL
Antithyroid peroxidase antibody negative
Thyroid 123 scintigraphy shows decreased uptake of radioactive iodine. Which of the following
is the most appropriate next step in management?
A) Metoprolol therapy
B) Prednisone therapy
OC) Propylthiouracil therapy
D) 1311 ablation
OE) No further management is indicated
25. A 42-year-old woman comes to the office 2 months after noticing a mass in her left breast.
There is no personal or family history of serious illness. Her last Pap smear 2 years ago and
mammography 1 year ago showed no abnormalities. Menses occur at regular 28-day intervals
and last 3 days. Her last menstrual period
was 2 weeks ago. She is sexually active with one male partner and uses an oral contraceptive.
She drinks four cups of coffee daily. Examination shows a 3-cm, firm, irregular mass in the
upper outer quadrant of the left breast. Examination of the right breast shows no abnormalities.
In addition to mammography, which of the
following is the most appropriate next step in management?
A) Discontinue oral contraceptive and reexamine in 1 month
OB) Perform breast biopsy
C) Perform breast MRI
D) Recommend discontinuing caffeine use
OE) Reexamine in 1 month
26. A 47-year-old woman comes to the physician because of heavy menstrual flow for 7 days.
Menses usually occur at regular 28-day intervals and last 4 to 5 days. She has not had
intermenstrual or postcoital bleeding. She has no history of serious illness and takes no
medications. Bimanual examination shows a uterus
consistent in size with a 20-week gestation. A Pap smear shows no abnormalities.
Ultrasonography of the pelvis shows an enlarged uterus with three solid, well-circumscribed
masses within the myometrium that are homogeneous in appearance. The largest mass is 7 cm
in diameter. Results of an endometrial biopsy
specimen show proliferative endometrium. Which of the following is the most likely diagnosis?
O A) Adenocarcinoma
B) Adenomyosis
C) Endometrial hyperplasia
D) Endometritis
O E) Leiomyomata uteri
27. A 37-year-old nulligravid woman comes to the physician because she has been unable to
conceive for 6 months. She has been married for 18 years and discontinued her oral
contraceptive 6 months ago. She is an attorney, and she and her husband have sexual
intercourse twice monthly. Menses occur at regular 28-day
intervals. She feels well and has no history of serious illness. Physical examination shows no
abnormalities. Which of the following is the most appropriate next step in management?
A) Marital therapy to increase frequency of sexual intercourse
OB) Use of an ovulation predictor kit to time sexual intercourse
C) Measurement of serum testosterone concentrations of the patient and her husband
D) Sildenafil therapy for the patient's husband
E) Intrauterine insemination during ovulation
28. A 32-year-old woman, gravida 2, para 2, comes to the phy sician for a routine health
maintenance examination. At her last examination 1 year ago, a Pap smear and screening test
results for sexually transmitted diseases were negative. Her two children, ages 6 and 4 years,
were delivered by cesarean section. Her first child
weighed 4394 g (9 lb 11 oz) at birth, and her second child weighed 4593 g (10 lb 2 oz) at birth.
The patient is sexually active with her husband and uses an oral contraceptive. She is 157 cm
(5 ft 2 in) tall and weighs 72 kg (158 lb), BMI is 29 kg/m2 Her temperature is 37°C (98.6°F),
pulse is 84/min, respirations are 18/min,
and blood pressure is 130/75 mm Hg. Physical examination shows no other abnormalities.
Which of the following is the most appropriate screening test at this visit?
A) Measurement of fasting serum glucose concentration
B) Measurement of serum thyroid-stimulating hormone concentration
C) Urinalysis
D) Mammography
E) Transvaginal ultrasonography for endometrial thickness
28. A 32-year-old woman, gravida 2, para 2, comes to the physician for a routine health
maintenance examination. At her last examination 1 year ago, a Pap smear and screening test
results for sexually transmitted diseases were negative. Her two children, ages 6 and 4 years,
were delivered by cesarean section. Her first child
weighed 4394 g (9 lb 11 oz) at birth, and her second child weighed 4593 g (10 lb 2 oz) at birth.
The patient is sexually active with her husband and uses an oral contraceptive. She is 157 cm
(5 ft 2 in) tall and weighs 72 kg (158 lb), BMI is 29 kg/m2 Her temperature is 37°C (98.6°F),
pulse is 84/min, respirations are 18/min,
and blood pressure is 130/75 mm Hg. Physical examination shows no other abnormalities.
Which of the following is the most appropriate screening test at this visit?
A) Measurement of fasting serum glucose concentration
B) Measurement of serum thyroid-stimulating hormone concentration
C) Urinalysis
D) Mammography
E) Transvaginal ultrasonography for endometrial thickness
29. A 47-year-old woman comes to the physician for a routine examination. Two years ago, she
underwent total thyroidectomy for medullary cancer, her postoperative course was
uncomplicated, but she did not keep her appointments for follow-up examinations. She says she
feels well. She has never had radiation therapy.
There is no family history of thyroid disease. Her only current medication is levothyroxine. Vital
signs are within normal limits. Examination of the neck shows a well-healed surgical scar; there
are no masses or cervical lymphadenopathy. The remainder of the examination shows no
abnormalities. Which of the following
serum measurements is most appropriate to order to determine if this patient's cancer is in
remission?
OA) CA 19-9
OB) Calcitonin
C) Chromogranin A
D) Thyroglobulin
E) Thyroid-stimulating hormone
30. A 54-year-old woman comes to the office because of a 1-day history of intermittent episodes
of severe, sharp pain in the right side of her back that occur every 30 minutes. The episodes are
associated with nausea; the patient has had two episodes of nonbloody vomiting since her
symptoms began. During the past month
she has had fatigue, generalized aches, and difficulty concentrating on simple tasks. She has
had painful urination during the past 2 days. She has osteoarthritis treated with ibuprofen as
needed. Her temperature is 37°C (98.6°F), pulse is 100/min, respirations are 20/min, and blood
pressure is 146/90 mm Hg. The patient is
in moderate distress and shifts around on the examination table. No murmurs are heard. Bowel
sounds are normal. Abdominal examination shows tenderness to percussion at the right flank.
There is no hepatosplenomegaly. Laboratory studies show:
Hemoglobin 12 g/dL
Hematocrit
38%
Color
clear yellow
Leukocyte count 5100/mm3
PH
7.0 (N=4.5-7.8)
Platelet count 204,000/mm3 Specific gravity 1.018 (N=1.003–1.029)
Serum
Blood
grossly positive
Na-
142 mEq/L
Glucose
negative
K
5 mEq/L
Bilirubin
negative
CI-
100 mEq/L
trace
HCO,
25 mEq/L
Ketones
negative
Ca2+
12 mg/dL
Nitrites
negative
Urea nitrogen 14 mg/dL
Leukocyte esterase negative
Glucose
92 mg/dL
Urobilinogen 0.2 mg/2 hr (N=0.1-1.1)
Creatinine 0.9 mg/dL
Protein
X-ray of the abdomen shows a 4-mm calcified calculus in the right ureteropelvic junction. There
is no hydronephrosis. Which of the following is the most appropriate additional study at this
time?
A) CT scan of the abdomen
B) Measurement of serum intact parathyroid hormone concentration
C) MRIs of the brain and abdomen
D) Urine culture
E) No further testing is indicated
31. A 67-year-old woman is brought to the emergency department by her daughter because of a
2-day history of progressive fatigue, confusion, shortness of breath with exertion, and difficulty
breathing when she lies flat. At night, she has had the sudden need to run to an open window
for air. She had a myocardial infarction 2
years ago but has had no recent chest pain. Her medications are metoprolol, lisinopril,
spironolactone, and furosemide. The daughter says that she does not know if her mother has
taken her medications during the past 2 days. On arrival, the patient appears ill and in mild
respiratory distress, she is ashen. Her temperature
is 36.4°C (97.6°F), pulse is 102/min and regular, respirations are 24/min, and blood pressure is
82/46 mm Hg. Pulse oximetry on room air shows an oxygen saturation of 89%. Examination
shows jugular venous distention. Crackles are heard throughout the lower lung fields. On
cardiac examination, an S. gallop is heard.
There is 2+ pitting edema at the ankles. Intravenous administration of which of the following is
the most appropriate next step in pharmacotherapy?
A) Ceftriaxone
B) Dopamine
C) Esmolol
D) Heparin
E) Nitroprusside
33. A study is being conducted to investigate the association between childhood exposure to
secondhand cigarette smoke and risk for developing lung disease as an adult in individuals who
have never smoked. A total of 500 adults with lung disease and 500 adults with no history of
lung disease are enrolled in the study
participants in both groups are interviewed about exposure to secondhand cigarette smoke
during childhood. Which of the following is the most appropriate measure of association for the
investigators to report?
A) Absolute risk
B) Attributable risk
C) Hazard ratio
OD) Odds ratio
E) Relative risk
34. A 2608-g (5-lb 12-oz) newborn is delivered at term to a 32-year-old primigravid woman who
received no prenatal care. Examination of the newborn shows generalized hypertonia.
Meconium testing for cocaine metabolites is positive. A review of the literature shows that this
screening test has a sensitivity of 100% and a
true-negative rate of 90%. Which of the following is necessary to calculate the positive
predictive value of the test?
A) Incidence of no prenatal care
B) Negative predictive value of the test
OC) Prevalence of cocaine use by pregnant women in the community
D) Rate of newborn testing for cocaine
E) Specificity of the test
35. A randomized placebo-controlled trial is initiated to investigate the use of a new medication
for the treatment of hypertension. The trial enrolled 1000 men and women ages 40 years and
older; these patients are equally randomized to either the new medication or a placebo. The
primary outcome was established as
cardiovascular death or morbidity. After 1 year, 4.2% of patients taking the new medication
experienced the primary outcome as determined by hospital records, compared with 3.4% of
patients taking placebo (P= 52). The investigators then reanalyze the outcomes for only African
American study patients. Of the 250 African
American patients in the study, 2.4% of those taking the new medication experienced the
primary outcome, compared with 6.0% in the placebo group (P=.03). The investigators conclude
that the medication is effective only for African American patients. Which of the following is the
primary threat to the validity of this
conclusion?
A) Inadequate sample size
B) Post hoc analysis
C) Recall bias
D) Sampling bias
OE) Volunteer bias
36. A 37-year-old man comes to the physician because of a 1-week history of pain with
swallowing. He received the diagnosis of AIDS 4 years ago, but he has not been able to tolerate
highly active antiretroviral therapy. He currently takes trimethoprim-sulfamethoxazole. Vital signs
are within normal limits. Examination shows a
few white plaques over the pharynx. No other abnormalities are noted. Laboratory studies show
a CD4+ T-lymphocyte count of 50/mm3 (Normal500) and a plasma HIV viral load of 50,000
copies/mL. Which of the following is the most appropriate pharmacotherapy?
A) Acyclovir
B) Amphotericin B
OC) Ciprofloxacin
OD) Fluconazole
E) Foscarnet
37. A 20-year-old man is brought to the physician by his parents because of auditory
hallucinations and bizarre behavior over the past year. He recently dropped out of college and
moved back home with his parents. He has not attempted to find a job. He says that he has
been feeling strange, "like being in a dream," and talks to
his great-great-grandfather who died 50 years ago. He was diagnosed with hepatitis A 2 years
ago after an episode of jaundice, and he has been treated with thyroxine for hypothyroidism for
the past 6 months. His pulse is 68/min, respirations are 10/min, and blood pressure is 100/70
mm Hg. Physical examination shows
hyperreflexia of the lower extremities and mild resting tremor of the upper extremities. On
mental status examination, his voice is monotonous, his face is immobile, and he seems very
anxious. He stares at the physician and barely answers questions. Serum studies show:
Total bilirubin
1 mg/dL
Thyroid-stimulating hormone 1 pU/mL
AST
21 U/L
ALT
20 U/L
Urine toxicology screening is negative. Which of the following is the most likely diagnosis?
A) Amyotrophic lateral sclerosis
B) Bipolar disorder
C) Hepatic encephalopathy
D) Major depressive disorder with psychotic features
E) Parkinson disease
F) Psychotic disorder due to a general medical condition
G) Schizophrenia
H) Substance-induced psychotic disorder
38. A 62-year-old man comes to the physician because of sadness and difficulty sleeping since
his wife of 35 years died of breast cancer 7 months ago. He began grieving for her before she
died, but the grief has become more intense since her death. During this period, he has had
loss of appetite and is no longer interested in
golfing and having dinner with friends. He has started to believe that he will never feel better
and sees himself as worthless. Physical examination shows no abnormalities. Mental status
examination shows a depressed mood and sad affect. He says that life is no longer tolerable,
but he has not thought about killing himself.
Which of the following is the most likely diagnosis?
A) Acute stress disorder
B) Adjustment disorder with depressed mood
OC) Dysthymic disorder
D) Major depressive disorder
E) Normal bereavement
39. A 27-year-old man is brought to the emergency department by police 1 hour after they found
him lying in the street and proclaiming himself to be invulnerable to death because he is "the
Antichrist." He is not able to give a coherent history, but imipramine and thioridazine bottles are
found in his pocket. He appears
somnolent. His pulse is 88/min, respirations are 20/min, and blood pressure is 110/70 mm Hg.
Physical examination shows red, dry skin and dilated pupils. Serum electrolyte concentrations
are within the reference ranges. An ECG shows a prolonged QT interval. The patient is admitted
to the intensive care unit. Which of
the following is the most appropriate next step in diagnosis?
A) Measurement of blood alcohol concentration
B) Measurement of serum imipramine concentration
C) Measurement of urine porphobilinogen concentration
D) MRI of the brain
E) Urine culture
40. A 32-year-old woman comes to the physician because of weakness in her right leg that
began 1 week ago and has resolved over the past 24 hours. She is currently asymptomatic.
Two years ago, she had tingling in both hands and an "electric-like" sensation down her spine
when she flexed her neck; her symptoms resolved after 3 weeks. Six months ago, she had
visual loss in her right eye and pain with eye movement. Examination at that time showed a
right central scotoma that resolved after 2 weeks. She has a history of mitral valve prolapse.
She takes no medications. Her pulse is 64/min and regular, and blood pressure is 100/60 mm
Hg. Cardiac examination shows a systolic click. Funduscopic examination shows mild pallor of
the right optic disc; there is no afferent pupillary defect. Muscle strength is normal.
Deep tendon reflexes are 2+ in the upper extremities and 3+ in the lower extremities. Babinski
sign is present on the right. An MRI of the brain is shown. Which of the following is the most
appropriate treatment for this patient?
A) Stress management
B) Amitriptyline therapy
C) Aspirin therapy
D) Dexamethasone therapy
OE) Interferon beta therapy
F) Propranolol therapy
G) Warfarin therapy
OH) Radiation therapy
41. A 77-year-old man with hypertension comes to the physician for a routine examination. His
medications are metoprolol and simvastatin. His father died of a cerebral infarction. The patient
smoked one pack of cigarettes daily for 25 years but quit 10 years ago. He walks 2 miles daily.
He is 180 cm (5 ft 11 in) tall and weighs
82 kg (180 lb); BMI is 25 kg/m2 His pulse is 70/min, and blood pressure is 140/90 mm Hg.
Examination shows no abnormalities. Serum studies show a cholesterol concentration of 210
mg/dL. Results of a recent cardiac stress test are negative. Which of the following is the
strongest predisposing factor for cerebral infarction
in this patient during the next 5 years?
A) Age
B) Family history
C) Hyperlipidemia
OD) Lack of antiplatelet therapy
E) Smoking history
42. A 57-year-old man comes to the physician because of a 10-year history of an intermittent
rash on his face. The rash worsens with cold weather and stress. Examination shows a scaly,
erythematous rash in several areas around the nose and eyebrows. There is dandruff from the
scalp. Which of the following is the most likely
diagnosis?
O A) Acne rosacea
B) Acne vulgaris
C) Herpesvirus infection
D) Psoriasis
E) Seborrheic dermatitis
43. A 67-year-old man comes to the physician 1 hour after a 5-minute episode of moderate
left-sided chest pain that began while he was unloading groceries. The pain radiated to the left
jaw. During the past 3 months, he also has had progressive shortness of breath when lying
down. He has a 1-year history of decreasing
stamina; he now often has to stop and rest while taking out the trash. He has well-controlled
hypertension, well-controlled type 2 diabetes mellitus, and chronic bronchitis. His medications
are hydrochlorothiazide, metoprolol, glipizide, and a nebulized albuterol inhaler as needed. He
smoked one and one-half packs of
cigarettes daily for 40 years but quit 10 years ago. His temperature is 37.2°C (99°F), pulse is
100/min, respirations are 16/min, and blood pressure is 100/58 mm Hg. Pulse oximetry on room
air shows an oxygen saturation of 90%. Examination shows jugular venous distention. Crackles
are heard at both lung bases. On
cardiac examination, there is an Szand a grade 2/6 harsh, late-peaking, systolic murmur heard
best at the upper right sternal border. There is 2+ pitting edema of the lower legs. Which of the
following is the most appropriate next step in diagnosis?
A) Coronary angiography
OB) Echocardiography
C) Exercise stress test
D) MR aortography
OE) Spiral CT scan of the chest
44. A 27-year-old man is brought to the emergency department 50 minutes after he sustained a
gunshot wound to his right leg. He was injured while attempting to flee a crime scene.
Paramedics report significant bright red bleeding that was controlled with pressure dressings at
the scene. On arrival, the patient is conscious
and responsive. His pulse is 100/min and regular, respirations are 18/min, and blood pressure is
100/85 mm Hg. Examination shows a single gunshot entry and exit wound in the right thigh with
moderate oozing of bright red and dark blood but no pulsatile bleeding. The mid thigh is
edematous. The right femoral pulse is
palpable; no popliteal or pedal pulses are palpable. The right ankle brachial index is 0.5 (N>1) at
the dorsalis pedis. The remainder of the examination shows no abnormalities. In addition to
intravenous fluid administration, which of the following is the most appropriate next step in
management?
OA) Arterial ligation
B) CT scan of the right lower extremity
C) Endovascular stent placement
OD) Intraoperative angiography
E) Intravenous administration of heparin
45. A 33-year-old woman, gravida 2, para 1, at 35 weeks' gestation is admitted to the hospital
because of a 1-hour history of moderate, bright red vaginal bleeding. During this time, she has
saturated two sanitary pads. She has not felt contractions or loss of amniotic fluid and reports
good fetal movement. She has received no
prenatal care. Her first pregnancy ended in cesarean delivery at term because of fetal
bradycardia. She is unemployed and lives in a local homeless shelter. She has a history of
cocaine use but has not used cocaine during the past 2 months. Her temperature is 37°C
(98.6°F), pulse is 100/min and regular, and blood
pressure is 90/60 mm Hg. The abdomen is nontender and consistent in size with a 35-week
gestation. External fetal monitoring shows a heart rate of 150/min with moderate variability,
several spontaneous accelerations, and occasional variable decelerations. An external
tocometer shows irregular uterine contractions.
Examination shows a small amount of bright red blood at the perineum. Which of the following is
the most appropriate next step in management?
A) Apt test
B) Contraction stress test
C) Digital cervical examination
OD) Immediate cesarean delivery
E) Pelvic ultrasonography
46. A 3-year-old girl is admitted to the hospital because of a 7-day history of fever and iritability.
Treatment with acetaminophen has not resolved her symptoms. During the past 2 days, she has
had a decreased appetite but a normal fluid intake. She appears irritable. Her temperature is
38.7°C (101.7°F). Examination shows
nontender, bilateral, cervical lymphadenopathy. The sclerae are injected without exudates.
There is an erythematous papular rash over the perineum. Examination of the hands and feet
shows erythema and tenderness. Her hemoglobin concentration is 10.2 g/dL, and platelet count
is 285,000/mm3 Urinalysis shows 50
WBC/hpf and no bacteria. Which of the following is the most appropriate pharmacotherapy?
A) Oral ibuprofen
B) Oral warfarin
C) Intramuscular penicillin
OD) Intravenous immune globulin
E) Intravenous prednisone
47. An 87-year-old woman with metastatic ovarian cancer has poorly controlled pain. Her
cancer has been unresponsive to chemotherapy. Current medications include long-acting oral
morphine, short-acting morphine as needed, and docusate. Her family is having difficulty caring
for her at home. The patient's oldest daughter
calls the physician to discuss her mother's care. She says, "We want my mother to receive
hospice care at home, but no one wants her to die at home. Can she still have hospice
services?" Which of the following is the most appropriate response?
A) "No, but we can try to arrange for some other type of home service."
B) "No. It is important that hospice patients die at home."
C) "No. The patient cannot be enrolled in the hospice program if the patient or family prefer that
death not take place at home."
D) "Yes, but we would have to transfer her to a nursing home to enroll her in the hospice
program."
E) "Yes. Hospice can provide home-based care and attempt to transfer the patient to another
site before death."
48. Two days before undergoing removal of four molar teeth, a 27-year-old man comes to the
physician for therapy to prevent excessive bleeding. He has a lifelong history of mild mucosal
bleeding. His sister has a history of similar bleeding. Examination shows no abnormalities.
Laboratory studies show
Platelet count
225,000/mm3
Bleeding time
11 min
Prothrombin time
11 sec (INR=1)
Partial thromboplastin time 47 sec
Factor VIII
15% of normal (reference range: 50%-150% of normal)
Which of the following is the most likely explanation for these findings?
A) Antiphospholipid antibody syndrome
B) Hemophilia A
C) Inhibitor to factor VIII
D) Lupus anticoagulant
E) von Willebrand disease
49. A 23-year-old woman comes to the physician for a follow-up examination. Six months ago,
she underwent splenectomy because of injuries sustained in a motor vehicle collision. Her
postoperative course has been uncomplicated. She has a headache once monthly and takes
acetaminophen as needed. Immunizations are
up-to-date. Vital signs are within normal limits. Examination shows no abnormalities except for a
well-healed scar. She asks if any precautions are required before her dental appointment in 1
week. Which of the following is the most appropriate recommendation?
A) Postponement of dental work for 6 months
B) Measurement of serum complement concentrations
C) Influenza virus vaccine
OD) Penicillin therapy prior to teeth cleaning
E) No precautions are necessary
50. A 67-year-old man comes to the physician because of a 2-month history of progressive
shortness of breath on exertion, muscle pain, fatigue, generalized weakness, and constipation.
He also has had a decreased appetite, resulting in a 7-kg (15-Ib) weight loss during this period.
His current medications are
hydrochlorothiazide for hypertension and laxatives. He smoked one pack of cigarettes daily for
40 years but quit 5 years ago. His temperature is 37.5°C (99.5°F), pulse is 85/min, respirations
are 22/min, and blood pressure is 164/92 mm Hg. There is no jugular venous distention. Breath
sounds are decreased, and scattered
rhonchi are heard over the right upper lung field. Cardiac examination shows no abnormalities.
Laboratory studies show:
Hemoglobin
14.2 g/dL
Leukocyte count 11.500/mm3 with a normal differential
Serum
Na+
140 mEq/L
K+
3.5 mEq/L
CI-
100 mEq/L
HC0 -
25 mEq/L.
Ca 2+
13.3 mg/dL
Urea nitrogen
22 mg/dL
Glucose
72 mg/dL
Creatinine
0.8 mg/dL
Alkaline phosphatase 100 U/L
A chest x-ray shows a mass in the upper lobe of the right lung that extends into the hilum.
Which of the following is the most likely cause of these laboratory findings?
A) Adverse effect of hydrochlorothiazide
B) Bone metastases from lung cancer
OC) Paraneoplastic syndrome
D) Parathyroid carcinoma
E) Primary hyperparathyroidism
2. A 2-month-old boy is admitted to the hospital because of a 1-day history of coughing, rapid
breathing, and poor feeding. He had a temperature of 39°C (102.2°F) 4 hours ago. He has not
had a runny nose. One month ago, he was admitted to the hospital for 10 days for treatment of
orbital cellulitis. He was born at term after
an uncomplicated pregnancy and spontaneous vaginal delivery. Results of maternal prenatal
screening, including HIV serologic studies, were negative. On admission, the patient is alert and
has grunting respirations and mild intercostal retractions. His temperature is 37.9°C (100.2°F),
pulse is 170/min, respirations are
86/min, and blood pressure is 74/48 mm Hg. Pulse oximetry on 40% oxygen via nasal cannula
shows an oxygen saturation of 92%. Physical examination shows a supple neck with no
adenopathy. Breath sounds are decreased bilaterally. Diffuse crackles are heard in both lungs.
Cardiac examination shows a regular rhythm.
S, and S, are normal. A grade 1/6, systolic ejection murmur is heard at the upper left sternal
border. The abdomen is soft and nontender. The liver edge is firm and palpated 4 cm below the
right costal margin, and the spleen tip is palpated 2 cm below the left costal margin. Laboratory
studies show:
Hemoglobin
9.6 g/dL
Hematocrit
29%
Leukocyte count
9000/mm3
Segmented neutrophils 70%
Bands
Eosinophils
3%
Lymphocytes
20%
Monocytes
3%
Platelet count
220.000/mm3
CD4+ T-lymphocyte count 150/mm3 (Normal 1750)
Serum
IgA
IgG
10 mg/dL
IgM
10 mg/dL
<10 mg/dL
A chest x-ray shows diffusely hazy lung fields, no cardiomegaly, and a normal mediastinal
silhouette. Which of the following is the most likely underlying diagnosis?
OA) Chédiak-Higashi syndrome
B) Chronic granulomatous disease
C) HIV infection
OD) Severe combined immunodeficiency (SCID)
E) Thymic-parathyroid dysplasia
F) X-linked agammaglobulinemia
3. A 55-year-old woman comes to the physician because of a 6-week history of easy bruising.
Ten years ago, she underwent local resection and chemotherapy for cancer of the right breast.
She takes no medications. Examination shows multiple cutaneous ecchymoses over the body,
primarily over all the extremities. The spleen
is not palpable. Her hemoglobin concentration is 12 g/dL, hematocrit is 34%, and platelet count
is 34,000/mm3. Which of the following is the most likely diagnosis?
OA) Acute myelogenous leukemia
B) Aplastic anemia
C) Immune thrombocytopenic purpura
D) Metastatic breast cancer
E) Secondary hypersplenism
5. A 47-year-old woman comes to the physician because of difficulty sleeping for 2 months.
During this period, she has been awakening at 3 am and remaining awake worrying about work,
even though she is tired. She has unstable angina pectoris and required placement of one
coronary artery stent 6 months ago and a
second stent 4 months ago. Her chest pain started to recur 1 month ago, and she now has it
almost daily. Evaluation shows no organic cause for her recurring chest pain. She works as an
attorney for a large firm and reports that she has been given more responsibility during the past
year. For the past month, she has not
enjoyed her work as much as she previously did. She forces herself to go to work and has
difficulty paying attention while she is there. She is a gourmet cook but no longer prepares
meals because she does not enjoy the taste of the food. Physical examination shows no
abnormalities. On mental status examination, she has
a sad and worried mood and a reactive affect. She is alert and oriented to person, place, and
time. She states that she frequently becomes despondent, especially when she thinks about her
heart disease. Which of following is the most likely diagnosis?
A) Adjustment disorder
B) Dysthymic disorder
OC) Generalized anxiety disorder
D) Major depressive disorder
E) Primary insomnia
6. A 43-year-old woman is brought to the emergency department 30 minutes after the onset of
tonic-clonic movements of all extremities at home. Her son says she has a 10-year history of
chronic back pain, for which she takes oxycodone and diazepam. She previously worked as a
nurse but has not worked during the past
5 years because of the pain. She lives with her two adult sons. On arrival, the patient is
observed speaking clearly while the tonic-clonic movements continue. The episode ends 20
minutes after arrival; following the episode, the patient is alert and fully oriented with intact
memory. Vital signs are within normal limits. Pulse
oximetry on room air shows an oxygen saturation of 99%. Examination shows no evidence of
incontinence or tongue biting during the episode. Which of the following is the most appropriate
next step in management?
A) Addition of phenytoin to the medication regimen
B) EEG
C) Increasing the dosage of diazepam
D) Lumbar puncture for examination of cerebrospinal fluid
E) Reassurance that she did not have a neurologic event
8. A 27-year-old woman comes to the physician because of a 1-year history of daily headaches
involving her entire head. The headaches are worse in the afternoon, sometimes throbbing, and
are associated with nausea and sensitivity to light. The headaches are worse during menses.
She takes four to six tablets of
acetaminophen with caffeine daily, which provides partial relief. She takes no other medications.
She has smoked one-half pack of cigarettes daily for 10 years and drinks four beers weekly.
She works the night shift at an industrial plant. Vital signs are within normal limits. Neurologic
examination shows no focal findings.
Which of the following is the most appropriate treatment for this patient's headaches?
OA) Switching to daytime shift work
B) Cessation of alcohol consumption
C) Cessation of smoking
D) Discontinuation of acetaminophen with caffeine
E) Oral contraceptive therapy
9. A 25-year-old woman, gravida 1, para 1, delivers a 2268-9 (5-lb) male newborn at 38 weeks'
gestation. The mother received no prenatal care. She has no history of serious illness and takes
no medications. During pregnancy, she drank four to six glasses of wine daily and used
cocaine weekly. A photograph of the newborn is shown. Which of the following during the
mother's pregnancy would have significantly decreased the risk of this fetal outcome?
OA) Abstinence from alcohol
B) Abstinence from cocaine
C) Copper supplementation
OD) Folic acid supplementation
E) Vitamin B, (thiamine) supplementation
OF) Zinc supplementation
10. An 11-year-old boy is brought to the emergency department because of a 1-hour history of
severe pain and blurred vision in his right eye that began when he was hit in the eye with a
tennis ball. He says his eye was closed when he was hit. He has no history of serious illness
and takes no medications. He cries and
appears anxious but gradually calms down. Vital signs are within normal limits. Ophthalmologic
examination of the right eye shows mild periorbital edema; pooling blood in the anterior
chamber obscures the lower third of the iris and pupil. Blinking does not increase pain. Gaze is
conjugate and symmetric. The right pupil is
slightly larger and less reactive to light compared to the left pupil. There are no foreign bodies or
visual field defects. The remainder of the neurologic examination shows no focal findings. Which
of the following is the most likely diagnosis?
OA) Concussion
B) Corneal abrasion
OC) Hyphema
D) Orbital fracture
E) Retro-orbital bleeding
OF) Subconjunctival hemorrhage
11. A 15-year-old girl is brought to the physician 2 weeks after noticing multiple areas on her
chest and upper back that are lighter than the rest of the skin. She has mild itching over these
areas when she plays volleyball. A photograph of the lesions is shown. Which of the following is
the most likely diagnosis?
OA) Actinic keratosis
B) Atopic dermatitis
C) Contact dermatitis
D) Discoid lupus erythematosus
E) Impetigo
F) Rosacea
G) Seborrheic dermatitis
H) Tinea corporis
1) Tinea versicolor
J) Urticaria
K) Vitiligo
12. A 42-year-old woman with rheumatoid arthritis is admitted to the hospital because of a 3-day
history of fever, intermittent severe chest pain, mild abdominal pain, and malaise. She also has
a 1-week history of increasing fatigue. The chest pain worsens with inspiration, and she says
she now sleeps most comfortably while
leaning over her kitchen table. During the past week, she has had increased joint pain and
decreased appetite. She has not had vomiting or changes in bowel function. Her medications
are methotrexate, prednisone, and acetaminophen with codeine. She does not smoke
cigarettes, drink alcohol, or use illicit drugs. She is in
moderate distress. She is 165 cm (5 ft 5 in) tall and weighs 59 kg (130 lb); BMI is 22 kg/m2 Her
temperature is 38.6°C (101.4°F), pulse is 90/min, respirations are 22/min and shallow, and
blood pressure is 140/88 mm Hg. On examination, a pleural friction rub is heard over the lower
lobe of the right lung, and a pericardial
friction rub is heard over the anterior aspect of the left hemithorax. The spleen tip is palpated 2
cm below the left costal margin. There is no peripheral edema. There is thickening of the
synovial membranes and tenderness to palpation of the hands and feet. Laboratory studies
show:
Hemoglobin
11.1 g/dL
Hematocrit
33%
Mean corpuscular volume 88 um
Leukocyte count
1800/mm3
Segmented neutrophils 65%
Eosinophils
2%
Basophils
1%
Lymphocytes
30%
Monocytes
2%
Platelet count
155,000/mm3
Serum
Urea nitrogen
24 mg/dL
Creatinine
1.2 mg/dL
AST
12 U/L
ALT
24 U/L
Which of the following is the most likely diagnosis?
OA) Amyloidosis
B) Bacterial endocarditis
C) Cirrhosis
OD) Felty syndrome
E) HIV infection
F) Leukemia
OG) Tuberculosis
13. A 54-year-old man comes to the office because of a 4-month history of progressive
weakness and numbness of his left leg. He has a 5-year history of type 2 diabetes mellitus
treated with metformin. He has worked as a plumber for 34 years; he often crawls into and
works in tight spaces, and he frequently moves and lifts
heavy objects. On neurologic examination, muscle strength is 5-15 in the thigh adductor,
quadriceps, and tibialis anterior muscles on the left and intact on the right. Muscle strength is
intact in the iliopsoas, gluteus maximus, thigh abductor, hamstrings, and gastrocnemius
muscles bilaterally. Deep tendon reflexes are 1+ in the
left knee, 2+ in the right knee, and 2+ in the ankles. Sensation to light touch is mildly decreased
over the medial-distal aspect of the left lower extremity and medial aspect of the left foot.
Sensation otherwise is intact. Which of the following is the most likely diagnosis?
A) Common peroneal neuropathy
B) Femoral neuropathy
C) L4 radiculopathy
D) Obturator neuropathy
E) S1 radiculopathy
14. A 5-year-old girl with spastic diplegic cerebral palsy and gastroesophageal reflux disease is
brought to the office by her parents for a routine examination. She wears leg braces and
occasionally uses a wheelchair for ambulation. Since her last visit 6 months ago, she has had
intermittent shortness of breath when walking long
distances and has tended to lean toward the right. Her parents say she has had decreased
tolerance for sitting in her wheelchair. She was born at 31 weeks' gestation by cesarean delivery
to a 21-year-old woman, gravida 4, para 4; pregnancy was complicated by chorioamnionitis.
The patient has undergone soft-tissue
release for contractures in her hips and lower extremities. Her medications are baclofen and
lansoprazole. She is alert and fully oriented, and she answers questions appropriately. She is at
the 40th percentile for height and weight. Vital signs are within normal limits. Mild impingement
of the rib cage on the right iliac crest is
noted. Cardiopulmonary examination shows no abnormalities. Pelvic obliquity is present.
Examination of the upper extremities shows less spasticity and moderate fine motor control
when compared with the lower extremities. There is mild
tenderness to palpation of the right lower rib cage where it abuts the right iliac crest.
Patellar and Achilles deep tendon reflexes are 3+ bilaterally. Which of the following is the most
appropriate next step in diagnosis?
OA) Bone scan
OB) Electromyography and nerve conduction studies
OC) Gait analysis
D) MRI of the brain
E) X-rays of the spine
15. A 37-year-old woman comes to the physician for endocarditis prophylaxis prior to
undergoing tooth extraction. The patient reports mouth pain but otherwise feels well. Two years
ago, echocardiography showed mitral valve prolapse and trace mitral regurgitation. She has no
other history of serious illness and takes no
medications. Vital signs are within normal limits. Cardiac examination shows a normal S, and S,
there is no S, or S4, and no murmurs are heard. Which of the following is the most appropriate
oral antibiotic prophylaxis prior to this patient's dental procedure?
OA) Amoxicillin
B) Azithromycin
C) Clindamycin
OD) Moxifloxacin
E) No antibiotic prophylaxis is indicated
16. An 18-year-old man has the sudden onset of rapid, pounding heartbeats. A previous
episode occurred while he was drinking coffee and resolved abruptly after 30 minutes. His pulse
is 180/min, respirations are 15/min, and blood pressure is 105/85 mm Hg. Examination shows
no other abnormalities. Which of the following is
the most likely diagnosis?
OA) Atrial fibrillation
B) Multifocal atrial tachycardia
OC) Paroxysmal supraventricular tachycardia
D) Paroxysmal ventricular tachycardia
E) Premature atrial beats
F) Premature ventricular beats
G) Sinus tachycardia
OH) Third-degree atrioventricular block
17. A 66-year-old woman comes to the office because of a 6-month history of severe dizziness
and a spinning sensation that occurs when she looks up or turns her head quickly. She says she
fell 1 week ago during an episode. At that time, she had a brief episode of double vision. She
also has a 2-year history of moderate neck
pain, which she attributes to "muscle spasms", since her fall, her pain has become more severe.
She has hypertension and type 2 diabetes mellitus. Her medications are lisinopril and
metformin. Her mother had a cerebral infarction at the age of 70 years. The patient has smoked
one-half pack of cigarettes daily for 40 years.
Her pulse is 84/min, and blood pressure is 150/85 mm Hg. Examination shows no nystagmus.
Visual field testing shows no abnormalities. There are no cervical bruits. On palpation of the
posterior muscles of the neck, there is mild tenderness that increases when the neck is rotated
gently. Cardiopulmonary examination
shows no abnormalities. Muscle strength is 5/5 in all extremities. Which of the following is the
most likely explanation for this patient's symptoms?
A) Carotid artery dissection
OB) Cervical intervertebral disc disease
C) Glomus vagale tumor
D) Ligamentous injury of the cervical spine
E) Vertebral artery stenosis
18. An 18-year-old man is brought to the emergency department (ED) 15 minutes after he was
involved in a high-speed motor vehicle collision in which he was the unrestrained driver. The
airbags did not deploy on impact. The patient was found unresponsive at the scene by
emergency medical technicians, who intubated him
prior to transport. On arrival at the ED, he remains unresponsive. Medical history and
medications are unknown. Pulse is 100/min, respirations are 16/min, and blood pressure is
105/60 mm Hg. Pulse oximetry on an Fio of 1.0 shows an oxygen saturation of 100%. Glasgow
Coma Scale score is 3. Physical examination
shows contusions over the forehead and anterior chest and crepitus of the anterior left chest.
Femoral pulses are 1+ bilaterally. Two large-bore intravenous catheters are inserted and 1 L of
lactated Ringer solution is administered with no change in hemodynamics. Chest x-ray is
shown. Which of the following is the most
appropriate diagnostic study at this time?
A) CT angiography of the chest
B) MRI of the right shoulder
C) Needle thoracostomy
D) Pericardiocentesis
OE) Repeat chest x-ray with dedicated rib views
19. A 25-year-old woman at 32 weeks' gestation comes to the office because of a 1-month
history of "pressure" under her eyes, persistent stuffy nose, and yellow-green nasal discharge.
Her pregnancy has been uncomplicated. Temperature is 37.2°C (99.0°F), pulse is 82/min, and
respirations are 16/min. Pulse oximetry on
room air shows an oxygen saturation of 99%. Physical examination discloses tenderness to
palpation over the cheeks. Which of the following is the most appropriate initial step in
management for this patient?
A) Allergy testing
OB) Antibiotic therapy
C) Antihistamine therapy
D) Sinus ostial dilation
E) Reassurance that colds tend to last longer during pregnancy
20. A 72-year-old man comes to the office because of a 2-year history of progressive shortness
of breath that worsens with exertion. He also has a 1-year history of intermittent cough
productive of whitish sputum, primarily in the morning, there is no blood. He has not had chest
pain. He has hypertension and hyperlipidemia.
His medications are chlorthalidone, felodipine, potassium chloride, and simvastatin. He has
smoked one pack of cigarettes daily for 55 years. He is 183 cm (6 ft) tall and weighs 73 kg (160
lb); BMI is 22 kg/m2 His temperature is 37°C (98.6°F), pulse is 76/min, respirations are 18/min,
and blood pressure is 128/72 mm Hg.
Pulse oximetry on room air shows an oxygen saturation of 91%. Breath sounds are decreased
bilaterally; the lungs otherwise are clear to auscultation. On cardiac examination, S, and S, are
normal. A grade 2/6, systolic ejection murmur is heard at the upper right sternal border. There is
trace pedal edema bilaterally; there
is no cyanosis or clubbing. His hemoglobin concentration is 16 g/dL, and hematocrit is 49%. A
chest x-ray shows no infiltrates or effusions. An ECG shows no abnormalities. Which of the
following is the most appropriate next step in diagnosis?
A) CT scan of the chest
B) Echocardiography
OC) Nuclear cardiac stress scintigraphy
D) Pulmonary function testing
E) Ventilation-perfusion lung scans
21. A 62-year-old woman comes to the emergency department because of a 3-hour history of
right-sided chest pain. The pain is worse with coughing and deep breathing. She was
discharged from rehabilitation 1 month ago after an uncomplicated left knee replacement. Her
only medication is acetaminophen for pain. She is
165 cm (5 ft 5 in) tall and weighs 114 kg (252 lb); BMI is 42 kg/m2 Her temperature is 37.5°C
(99.5°F), pulse is 110/min, respirations are 20/min, and blood pressure is 126/70 mm Hg. Pulse
oximetry on room air shows an oxygen saturation of 92%. Cardiac examination shows an
accentuated Sz. The remainder of the
examination shows no abnormalities except for limited range of motion of the left knee. A chest
x-ray shows plate-like atelectasis in the right lower lung base. Which of the following is the most
appropriate next step in diagnosis?
A) Serum D-dimer assay
B) Gram stain and culture of sputum
O C) Spiral CT scan
D) Bronchoscopy with cytology
OE) Pulmonary angiography
22. A 37-year-old man comes to the physician for a follow-up examination 3 weeks after being
diagnosed with hepatitis C. He has a long-standing history of intravenous heroin use. He is
currently receiving methadone therapy for heroin addiction. He has smoked one and a half
packs of cigarettes daily for 20 years and drinks
seven glasses of wine weekly. Examination shows no abnormalities. Serum assays are positive
for hepatitis B antibody and negative for hepatitis B antigen. Which of the following is the most
appropriate recommendation to minimize the progression of liver disease in this patient?
OA) Abstinence from alcohol
B) Smoking cessation
C) Lamivudine (3TC) therapy
D) Discontinue methadone as a treatment option for heroin addiction
E) Hepatitis B vaccine
23. A 47-year-old woman comes to the physician for a follow-up examination. Two weeks ago,
she underwent laparoscopic Nissen fundoplication for Barrett esophagus secondary to
gastroesophageal reflux disease. Since the operation, she has had upper abdominal bloating
after eating. She also has had a sensation of meat
sticking in her esophagus that resolves spontaneously after 1 hour. She has been unable to
burp or vomit to relieve her discomfort. She has been following an unrestricted diet. She has not
had any changes in bowel habits. Current medications include oxycodone-acetaminophen. She
appears well. Her temperature is 36.8°C
(98.2°F), pulse is 76/min, respirations are 14/min, and blood pressure is 116/78 mm Hg.
Examination shows well-healing surgical incisions with mild tenderness to palpation. The
abdomen is scaphoid with no masses. Which of the following is the most appropriate next step
in management?
A) Dietary modification
B) Abdominal x-rays
C) Treatment with a prokinetic agent
D) Treatment with a proton pump inhibitor
E) Biopsy of the distal esophagus
F) Esophageal dilatation
G) Esophagomyotomy
24. A 53-year-old man with alcoholism comes to the physician because of a 5-month history of
loose stools and persistent swelling of his ankles. He is otherwise asymptomatic. He drinks at
least a pint" of bourbon daily. Vital signs are within normal limits. Cardiopulmonary examination
shows no abnormalities. Abdominal
examination shows distention and a positive fluid wave. Laboratory studies show:
Mean corpuscular volume 108 um
Serum
Urea nitrogen
7 mg/dL
Creatinine
0.9 mg/dL
Albumin
2.9 g/dL
Total bilirubin
1.6 mg/dL
AST
64 U/L
ALT
74 U/L
Additional laboratory studies are most likely to show which of the following serum electrolyte
concentrations?
Na+ (mEq/L)
K+(mEq/L)
A)
132
3.1
B)
132
4.9
C)
140
4
OD)
148
3.1
OE) 148
4.9
25. A previously healthy 6-year-old boy is brought to the emergency department because of
cramping abdominal pain and right-sided scrotal pain for 4 hours. He vomited once on the way
to the hospital. Examination shows a distended abdomen. Bowel sounds are decreased, and
there is diffuse tenderness to palpation with
involuntary guarding. The right hemiscrotum is slightly discolored with swelling and tenderness
superiorly. The left hemiscrotum is normal, the testicle is normal. Which of the following is the
most appropriate next step in management?
A) Transillumination of scrotum
B) Valsalva maneuver
C) Urinalysis
D) Doppler ultrasonography
E) CT scan
F) Antibiotic therapy
G) Operative procedure
26. A 56-year-old man with claudication is scheduled for angiography of the right femoral artery.
He has hypertension and type 2 diabetes mellitus treated with insulin. He smoked two packs of
cigarettes daily for 30 years until he stopped 5 years ago. He has no history of allergy to
radiopaque contrast agents. He has proteinuria
secondary to diabetes. Before angiography, which of the following should be administered to
prevent further renal injury?
A) Captopril
B) Corticosteroids
OC) Diphenhydramine
D) Increased dose of insulin
OE) 0.9% Saline
27. A 2-year-old girl is brought to the physician by her father because of a 4-day history of
vomiting and diarrhea. The patient has not voided urine during the past 24 hours. She has no
history of serious illness and receives no medications. Immunizations are up-to-date. She
appears tired but not in distress. Her pulse is
120/min, respirations are 24/min, and blood pressure is 110/60 mm Hg. Examination shows dry
mucous membranes. No other abnormalities are noted. Laboratory studies are most likely to
show which of the following findings?
OA) Red blood cell casts
OB) Serum potassium concentration of 6 mg/dL
OC) Urine fractional excretion of sodium equal to 0.5%
D) Urine osmolality of 300 mOsmol/kg H20
E) Urine specific gravity of 1.010
28. A 29-year-old woman, gravida 2, para 1, at 32 weeks' gestation is brought to the emergency
department 30 minutes after cutting her left index finger while cooking. She has been applying
pressure, but the bleeding has not stopped. She has no history of serious illness, and her only
medication is a prenatal vitamin.
Vaccinations were up-to-date at the birth of her first child 14 months ago. The patient's vital
signs, including oxygen saturation, are within normal limits. Examination shows a 2-cm
superficial laceration over the left index finger. Fetal heart tones are normal. The remainder of
the examination shows no abnormalities. The
laceration is sutured. Which of the following is the most appropriate vaccine to administer at this
time?
O A) Human papillomavirus
B) Inactivated influenza virus
C) Measles-mumps-rubella
D) Tetanus-diphtheria-acellular pertussis (Tdap)
E) No vaccine is indicated at this time
29. A 30-year-old primigravid woman at 26 weeks' gestation comes to the office for a routine
prenatal visit. Her pregnancy has been complicated by first-trimester vaginal bleeding. She is
otherwise healthy and takes no medications. Her pulse is 90/min, respirations are 20/min, and
blood pressure is 100/72 mm Hg. Examination
shows a fundal height of 27 cm. The fetal heart rate is 150/min. Laboratory studies show:
Hematocrit
33%
Leukocyte count
9500/mm3
Platelet count
220.000/mm3
O, Rh-negative
1-Hour glucose tolerance test 120 mg/dL (N<135)
Serum anti-D antibody screening positive 1:8
Blood group
The blood group of the fetus's father is unknown. Which of the following is the most likely
diagnosis?
A) Duffy sensitization
B) Folic acid deficiency
C) Gestational diabetes
D) Iron deficiency anemia
O E) Rh isoimmunization
30. A 37-year-old woman, gravida 2, para 1, at 15 weeks' gestation comes to the physician for
her first prenatal visit. She has felt well except for mild nausea for the past 4 weeks. She
delivered a healthy newborn 5 years ago, pregnancy and delivery were uncomplicated. She has
no history of serious illness or operative
procedures. Physical examination shows no abnormalities. The uterus is consistent in size with
a 13-week gestation. The patient inquires about amniocentesis but is concerned about any
adverse effects. If this patient undergoes amniocentesis, she is at increased risk for which of the
following complications?
A) Abruptio placentae
B) Meconium in the amniotic fluid
C) Polyhydramnios
OD) Premature rupture of membranes
E) Uterine rupture
31. A 49-year-old nulligravid woman with type 2 diabetes mellitus comes to the office because of
an 18-month history of increasingly heavy irregular menstrual periods. Today, she reports
vaginal spotting. Menses occur at 10- to 12-day intervals and last 5 to 7 days; she has to
change a regular tampon every 4 to 6 hours. She is
170 cm (5 ft 7 in) tall and weighs 104 kg (230 lb); BMI is 36 kg/m2 Pelvic examination shows a
slightly enlarged uterus without adnexal masses. Which of the following is the most appropriate
next step in diagnosis?
A) Complete blood count
B) CT scan of the pelvis
C) Determination of hemoglobin A10
OD) Endometrial biopsy
E) Measurement of serum progesterone concentration
32. A 27-year-old woman comes to the physician because of a 6-month history of decreased
bleeding with menses. She is otherwise asymptomatic. Menses occur at regular 28-day intervals
and last for 3 days with light spotting. Her only medication is an oral contraceptive for the past 5
years. Physical examination, including
pelvic examination, shows no abnormalities. Which of the following is the most appropriate next
step in management?
OA) Reassurance
B) Measurement of serum prolactin concentration
C) Progesterone challenge test
D) Pelvic ultrasonography
E) Estrogen supplementation
33. A 27-year-old man comes to the physician for a routine follow-up examination. He has a
22-year history of type 1 diabetes mellitus controlled with daily insulin. He measures his blood
glucose concentration once weekly. He has not developed diabetic complications. He does not
smoke or drink alcohol. He has an active
lifestyle. His pulse is 87/min, and blood pressure is 120/78 mm Hg. Cardiac examination shows
no abnormalities. Deep tendon reflexes are brisk and symmetric. His hemoglobin Ate is 6.5%.
Which of the following is the most appropriate screening recommendation at this time?
OA) Serum lipid studies
B) Liver function tests
C) Electromyography of the lower extremities
D) ECG
E) Ultrasonography of the pancreas
34. A 4-year-old boy is brought to the physician 3 hours after his mother removed a tick from his
scalp. She said that he was playing outside this morning. She washes his hair nightly and did
not notice the tick last night. He has had no symptoms. He has no history of serious illness. He
appears well developed and well
nourished and is in no acute distress. Examination shows no abnormalities. Which of the
following is the most appropriate next step in management?
A) Intramuscular ceftriaxone prophylaxis now
B) Measurement of serum Lyme (Borrelia burgdorferi) antibody concentration now and again in
2 weeks
C) Oral amoxicillin prophylaxis now
D) Oral doxycycline prophylaxis now
E) Reassurance and follow-up examination in 2 weeks
35. A 67-year-old woman comes to the physician because of a 10-day history of left buttock
pain that radiates down her left leg. She describes the pain as shooting and burning. She has a
2-year history of multiple myeloma poorly responsive to chemotherapy. Treatment with
oxycodone-acetaminophen (two tablets every 6
hours) has not completely relieved her pain, she rates it as a 5 on a 10-point scale. She lives
alone and can perform activities of daily living independently. Her temperature is 37°C (98.6°F),
pulse is 80/min, respirations are 14/min, and blood pressure is 125/85 mm Hg. Examination
shows no tenderness to palpation over the
lower spine. There is hyperesthesia to light touch over the left thigh. Muscle strength and
sensation are intact. Which of the following is the most appropriate pharmacotherapy?
A) Increase oral oxycodone-acetaminophen to four tablets every 6 hours
B) Add oral hydrocodone acetaminophen every 4 hours to the regimen
C) Add oral meperidine every 6 hours to the regimen
D) Add oral naproxen every 12 hours to the regimen
E) Add oral nortriptyline at bedtime to the regimen
36. A 57-year-old man is brought to the emergency department by police 40 minutes after he
was found wandering around a train station in a confused state. On arrival, he appears drowsy.
His temperature is 37°C (98.6°F), pulse is 72/min, respirations are 14/min, and blood pressure
is 135/80 mm Hg. Physical examination
shows bilateral nystagmus and a disconjugate lateral gaze. His gait is unsteady and broad
based. On mental status examination, he speaks in fragmented sentences and is often
incoherent. He can recall zero of three objects after 5 minutes. He is oriented to person but not
to place and time. Serum studies show:
140 mEq/L
K+
4 mEq/L
CI-
104 mEq/L
HC0 -
25 mEq/L
AST
70 U/L
ALT
64 U/L
y-Glutamyltransferase 55 U/L (N=0-30)
A urine toxicology screening is negative. His blood alcohol concentration is 0 mg/dL. Which of
the following is the most appropriate initial step in management?
A) Obtain collateral information
B) CT scan of the head
C) Chlordiazepoxide therapy
D) Donepezil therapy
E) Haloperidol therapy
OF) Vitamin B. (thiamine) therapy
37. A randomized controlled trial is conducted to examine the benefit of using graduated
compression stockings for prevention of deep venous thrombosis (DVT) in patients hospitalized
for conditions other than cerebral infarction. An additional agent, heparin or
low-molecular-weight heparin, is used for prophylaxis in some
patients. Results show
Group Type
Graduated Compression Additional Agent Rate of DVT (%)
Stockings
Study
Yes
13
2
Control
No
26
3.
Study
Yes
Yes
Control
No
Yes
1.
No
No
4
16
4
Which of the following best represents the relative risk reduction of DVT using graduated
compression stockings as a preventive measure compared with no prophylaxis?
A) 13%
B) 25%
C) 50%
D) 75%
E) 217%
38. A 68-year-old woman comes to the physician for an examination prior to undergoing total
right hip replacement. Preoperative ECG shows a Pwave area greater than 4 milliseconds x mV
in lead II, findings consistent with enlargement of the left atrium. A recently published study that
compares several ECG criteria for left
atrial enlargement (LAE) with a reference standard of cardiac CT scanning shows that this
criterion has a sensitivity of 73% and a specificity of 19%. Which of the following conclusions is
most appropriate based on these data?
A) Based on this criterion, 19% of patients with LAE will be correctly diagnosed with the
condition
B) Based on this criterion, 19% of patients without LAE will be incorrectly diagnosed with the
condition
C) Based on this criterion, 81% of patients with LAE will be correctly diagnosed with the
condition
D) Based on this criterion, 81% of patients without LAE will be incorrectly diagnosed with the
condition
E) Of all patients who do not meet this criterion for LAE, 19% will have the condition
OF) Of all patients who do not meet this criterion for LAE, 81% will have the condition
39. A study is conducted to evaluate the effects of smoking on birth weight at term. Four cohorts
of 10 pregnant patients each are identified on the basis of cigarette use. In cohort 1, patients
smoke less than one pack of cigarettes daily; in cohort 2, patients smoke one to less than two
packs of cigarettes daily, and in cohort 3,
patients smoke two or more packs of cigarettes daily during pregnancy. The control group
consists of patients who have never smoked cigarettes. Results of the study show the following
birth weights:
Cohort 1
Cohort 2
Cohort 3
Control Group
Mean
3155 g (6 lb 15 oz) 3028 g (6 lb 11 oz) 3015 g (6 lb 10 oz) 3310 g (7 lb 5 oz)
Standard Deviation
+ 350
+ 390
485
270
Which of the following is the most accurate interpretation of these results?
A) Smoking does not influence birth weight at term
B) The study is inconclusive because a type Il error is likely
C) There is a significant difference between cohort 1 and the control group
D) There is a significant difference between cohort 2 and the control group
E) There is a significant difference between cohort 3 and the control group
40. A 21-year-old college student comes to the physician for a follow-up examination. Seven
days ago, a biopsy specimen of a cervical lymph node showed Hodgkin disease, and 2 days
ago, an abdominal CT scan showed enlarged para-aortic lymph nodes. Examination today
shows no abnormalities except for cervical
adenopathy. He asks the physician, "What are my chances?" The physician recalls a recent
well-designed study that showed 80% of patients with stage III Hodgkin disease survive more
than 5 years. Which of the following is the most appropriate physician response?
A) "Hodgkin disease is a serious condition."
B) "I'm not sure. I will have to get back to you with an answer."
C) "It is hard to give you a reliable number. Try to take one day at a time."
D) "Tell me more about what you want to know."
E) "Twenty percent of patients like you only survive 5 years."
41. A 27-year-old man is brought to the emergency department by friends because of fever and
chills since beginning treatment with penicillin and azithromycin 4 hours ago for a sexually
transmitted disease. He has no history of serious illness and has no known allergies. On arrival,
he appears ill. His temperature is
39.4°C (103°F), pulse is 120/min, and blood pressure is 90/52 mm Hg. Examination shows
rigors. Which of the following is the most likely explanation for this patient's current symptoms?
OA) Acute bacterial sepsis
B) Acute HIV infection
C) Adverse effect of azithromycin
D) Adverse effect of penicillin
OE) Inadequate treatment of the infection
OF) Jarisch-Herxheimer reaction
42. A 27-year-old man comes to the physician because of a 3-month history of episodes of
palpitations, light-headedness, and tightness in his chest and throat. During the episodes, he
feels an impending sense of doom and fears that he might die or "go crazy." For the past 6
weeks, he has worried constantly that there is
something wrong with his heart and that the symptoms will recur. He recently ended a 3-year
relationship. He is a high school teacher. Physical examination shows no abnormalities. Mental
status examination shows an anxious affect. Which of the following is the most likely diagnosis?
A) Adjustment disorder with anxiety
B) Generalized anxiety disorder
C) Illness anxiety disorder
OD) Panic disorder
E) Somatization disorder
43. A 32-year-old woman comes to the physician because of a 4-day history of fever. She has
had swelling and redness of her left leg during this period. She has a history of chronic
lymphedema in both legs. She is 165 cm (5 ft 5 in) tall and weighs 107 kg (235 lb); BMI is 39
kg/m2 Her temperature is 38.5°C (101.3°F). The left
lower extremity is diffusely red and edematous from just below the knee to the ankle, with a
sharp demarcation separating the erythematous area from the normal skin at the knee. The
erythematous area is painful and hyperesthetic to touch. The left femoral nodes are enlarged
and painful. Which of the following is the most
likely cause of this patient's illness?
OA) Deep venous thrombosis
B) Escherichia coli infection
OC) Group A streptococcus infection
D) Necrotizing fasciitis
E) Superficial thrombophlebitis
44. A 3-month-old girl has had increased blood pressure measurements since admission to the
hospital for pneumonia and bronchiolitis 2 days ago. She was born at 26 weeks' gestation
because of premature labor and weighed 750 g (1 lb 10 oz) at birth. During the first 2 days of
life, an umbilical artery catheter was used to
monitor her blood pressure. She had several episodes of hypotension that resolved after
administration of fluid boluses. She remained in the neonatal intensive care unit for
management of respiratory distress and poor weight gain until the age of 10 weeks, when her
vital signs had stabilized and condition had improved. At
that time, she was discharged home on 0.5 L/min of oxygen. Her current medications are a
multivitamin with iron and budesonide. She appears well developed and well nourished but is in
respiratory distress. She weighs 3005 g (6 lb 10 oz). Her temperature is 37°C (98.6°F), pulse is
160/min, respirations are 70/min, and
blood pressure is 128/86 mm Hg. Pulse oximetry on room air shows an oxygen saturation of
92%. On pulmonary examination, crackles over the left lung base and scattered bilateral
expiratory wheezes are heard. On cardiac examination, a grade 1/6, systolic ejection murmur is
heard best at the upper left sternal border. The
abdomen is soft and nontender. Femoral pulses are 2+ bilaterally. Serum studies are most likely
to show which of the following sets of findings?
Renin Aldosterone
OA) Decreased decreased
B) Decreased increased
C)
Increased decreased
D
Increased increased
OE)
Normal normal
45. A 16-month-old girl is brought to the emergency department after the sudden onset of
difficulty breathing and cough at a birthday party. She has not had fever, vomiting, diarrhea,
runny nose, congestion, or a decrease in appetite or activity level. Immunizations are
up-to-date. There are no known sick contacts, and there is
no family history of serious illness. She appears healthy but is in moderate respiratory distress.
She is at the 50th percentile for length and weight. Her respirations are 40/min; other vital signs
are within normal limits. Pulse oximetry on room air shows an oxygen saturation of 96%.
Pulmonary examination shows minimal
subcostal retractions; no wheezes or crackles are heard. Breath sounds are mildly decreased
over the right upper lung fields. The remainder of the examination shows no abnormalities. An
AP chest x-ray shows normal findings. Which of the following is the most appropriate next step
in management?
OA) Bronchoscopy
B) Oral amoxicillin therapy
OC) Oral methylprednisolone therapy
D) Ventilation-perfusion lung scans
E) Observation for 48 hours
46. A 27-year-old nulligravid woman comes to the physician because of a 3-month history of
intermittent abdominal cramps and diarrhea. She has not had blood or mucus in her stool. She
reports that the symptoms usually occur after eating, are exacerbated by caffeine and stress,
and are relieved by bowel movements.
Changing her diet and avoiding dairy products have not improved her symptoms. She has two
to three bowel movements daily. Examination shows mild generalized tenderness and increased
bowel sounds. Which of the following is the most appropriate initial pharmacotherapy?
A) Ibuprofen
OB) Lactulose
O C) Loperamide
D) Mesalamine
E) Prednisone
47. A 24-year-old woman, gravida 2, para 2, comes to the physician requesting advice
regarding contraception. She states that she does not want to have any more children, but
neither she nor her husband wishes to undergo a permanent sterilization procedure at this time.
She requests a contraceptive method that has the
lowest failure rate for the first year of use. Her second child was born 6 weeks ago at term after
an uncomplicated pregnancy and delivery. Her infant is exclusively breast-fed. The patient has
no history of serious illness and takes no medications. She has not had sexual intercourse for 2
months. Examination shows no
abnormalities. Which of the following is the most appropriate contraceptive method for this
patient at this time?
OA) Use of a latex condom by her husband
B) Combination oral contraceptive therapy
OC) Progesterone-only oral contraceptive therapy
D) Vaginal contraceptive ring
OE) Levonorgestrel IUD
49. A 67-year-old woman comes to the physician because she has had a lesion on her right
eyelid for 3 months. She has no history of similar lesions. Examination shows a 2 x 1.5-cm
lesion on the lower lid of the right eye. A biopsy specimen of the lesion shows basal cell
carcinoma. Which of the following is the most
appropriate next step in management?
OA) Wide (1-cm) excision
B) Cryoablation
C) Chemotherapy
D) Radiation therapy
E) Mohs micrographic surgery
50. A 3-month-old boy is brought to the physician for evaluation of delayed growth. He was born
at term following an uncomplicated pregnancy and delivery. There was no cyanosis at birth. His
mother says that he feeds poorly and turns blue around the lips and fingers during feedings. He
frequently cries and becomes restless,
short of breath, and cyanotic. He is at the 20th percentile for length and 10th percentile for
weight. During the examination, he has paroxysms of crying during which he becomes cyanotic;
his color is normal at rest. A grade 3/6, systolic ejection murmur is heard at the left sternal
border. Which of the following is the most
likely diagnosis?
OA) Aortic stenosis
B) Atrial septal defect
C) Coarctation of the aorta
D) Patent ductus arteriosus
OE) Tetralogy of Fallot
F) Transposition of the great arteries
G) Truncus arteriosus
OH) Ventricular septal defect